Вы находитесь на странице: 1из 67

Geography- Subject wise compilation of 60 Day Plan- 2016

Q.1) Which of the following statements correctly explains a Cordillera?


a) It is a massive mountain range with two or more parallel mountain ranges.
b) It is a huge volcanic depression formed due to collapse of a volcano crater.
c) It is lake formed in mountain valleys.
d) It is lake formed in depressions of rocky deserts.
Q.1) Solution (a)
A Cordillera is a system of mountain ranges often consisting of a number of more or less
parallel chains. Eg. Rockies of North America, Himalayas of India etc.
Q.2) Consider the following statements about the Aravallis?
1. They are the oldest mountain ranges of India.
2. Aravalli is an example of block mountains.
3. Guru Shikhar is the highest peak of Aravalli range.
4. Aravalli hills lie perpendicular to the direction of monsoon winds causing heavy rainfall
in eastern Rajasthan.
Which of the above statements are correct?
a) 1 and 3
b) 2 and 4
c) 1,2 and 3
d) All of the above
Q.2) Solution (a)
Aravalli range is an old folded mountain range. They lie parallel to the monsoon winds
causing no interception hence very little rainfall.
Q.3) Consider the following statements:
1. When an air parcel is pushed up, first dry adiabatic cooling take place and then wet
adiabatic cooling take place.
2. During dry adiabatic cooling, air contains no water vapour.
3. The rate of dry adiabatic cooling is constant across the world, while rate of wet adiabatic
cooling constantly changes.
Which of the above statements are correct?
a) 1 and 2
b) 2 and 3
c) 1 and 3
d) All of the above
Q.3) Solution (c)

www.iasbaba.com

Geography- Subject wise compilation of 60 Day Plan- 2016

During dry adiabatic cooling air contains water vapour and during wet adiabatic cooling
air contains condensed water.
Dry adiabatic cooling is a physical phenomenon which occurs due to change in density as
the air parcel moves up. The rate is constant across the world. On contrary, during wet
adiabatic cooling, water starts condensing. Due to condensation, latent heat is released
which reduces the net rate of cooling. More the heat is released, slower the rate of
cooling becomes. Thats why it is not constant.

Q.4) Consider the following statements regarding Bhabhar plains:


1. It is the northern most tracts of Indian plains lying at the foothills of Siwaliks.
2. It contains fine alluvium deposited by the Himalayan streams.
3. It is very fertile and suitable for cultivation of crops.
Which of the above statements are incorrect?
a) All of the above
b) 1 only
c) 2 and 3
d) None of the above
Q.4) Solution (c)
Bhabar region lies along the foot of the Siwaliks from the Indus to the Tista. It comprises of
pebble-studded rocks in the shape of porous beds. It is due to porosity of the rocks; the
streams disappear and flow underground. The underground streams of the Bhabar reemerge on the surface and give birth to marshy area. The region is not much suitable for
agriculture.
Q.5) India and its neighboring countries are generally referred to as the Indian
Subcontinent. The reason behind this is:
a) Political dominance of India
b) Regions isolation from rest of the world
c) Vast size of the country
d) Typical trade links with the rest of the world
Q.5) Solution (b)
The region is isolated from rest of the world by Himalayan range in the north and Indian
ocean in the south. This isolation has evolved a unique culture of Indian subcontinent.
Clarification- India is considered a subcontinent because it has distinct geographical
features not shared by other Asian countries.

www.iasbaba.com

Geography- Subject wise compilation of 60 Day Plan- 2016


Q.6) Mt K2 is the highest peak of India. In which of the following mountain ranges does it
lie:
a) Karakoram Range
b) Greater Himalayas
c) Central Himalayas
d) Siwaliks
Q.6) Solution (a)
The height of Mt. K2 is 8611m from the sea level. It is the second highest mountain peak of
the world.
There is a controversy whether K2 is the highest peak of India or Kanchenjunga. K2 lies in
POK, but India consider it as Indian territory hence in all official records, K2 is the highest
peak of India.
Q.7) Which of the following statements are the major causes of drought in India?
1. High variability of the rainfall
2. Absence of forests resulting in rapid runoff and hence droughts
3. Rainfall is not distributed throughout the year
4. Amount of rainfall is inadequate
Select the correct code from the following:
a) 1,2 and 3
b) 2,3 and 4
c) 1,3 and 4
d) All of the above
Q.7) Solution (a)
The absolute amount of rainfall is high. Had it been distributed across the year it would have
been sufficient, but it is concentrated only in few months. Also the variability of rains at a
place is very high.
Q.8) Consider the following statements regarding Indias Shola forests:
1. They are evergreen mountain forests.
2. They are found only in high altitudes (>1500m) in tropical areas.
3. Trees are stunted and have low or no timber value.
Which of the above statements are correct?
a) 1 and 2
b) 2 and 3
c) 1 and 3
d) All of the above

www.iasbaba.com

Geography- Subject wise compilation of 60 Day Plan- 2016

Q.8) Solution (d)


The Sholas are a mosaic of mountane evergreen forests and grasslands. They are found only
in high altitude (>1500 metres asl) regions within the tropics, and are limited to the
southern part of the western ghats. They are characterised by undulating grassland patches,
interspersed with thickets of stunted evergreen tree species, and are home to a host of
endemic and endangered plants and animals. They are also vitally important in keeping
water cycles alive. They retain most of the rain they get over the monsoons, and release it
slowly through the year via a network of streams and rivers that eventually serve the needs
of a huge number of human settlements across south India.
Q.9) In which of the following regions there is a possibility of finding laterite soil?
a) Hot and humid
b) Hot and dry
c) Cold and dry
d) Cold and humid
Q.9) Solution (a)
Laterite soils are found in regions of very high rainfall. Due to rains the minerals get leached
to lower horizons. These soils are found in hot and humid areas.
Q.10) Consider the following statements regarding Deccan Trap:
1. It is a vast area of basaltic lava plateau.
2. The rocks are stratified into different layers showing multiple events of volcanism.
3. It is an area of large volcanic cones.
Which of the above statements are incorrect?
a) 1 and 2
b) 3 only
c) 1 and 3
d) None of the above
Q.10) Solution (b)
Basaltic lava has low viscosity and flow rapidly covering large area. It does not make volcanic
cones but results into volcanic shields.
Q.11) In which of the following coal mines do we get Anthracite coal?
1. Kalakot, Jammu and Kashmir
2. Jharia, Jharkhand
3. Neivelli, Tamilnadu

www.iasbaba.com

Geography- Subject wise compilation of 60 Day Plan- 2016


Select the correct code from the following:
a) 1 only
b) 2 and 3
c) 1 and 2
d) All of the above
Q.11) Solution (a)
Anthracite coal is found only at one place in India, Kalakot mine of J and K.
Jharia is famous for its bituminous coal while Neivelli has lignite
Q.12) The eastern and western coast of Indian peninsula has got contrasting features.
Which of the following contrasts are correct about the coastal plains?
1. Eastern coast is an example of submerging coast while western coast is an example of
emerging coast.
2. A number of rivers form deltas on the western coast, while it is not true for the eastern
coast.
3. As compared to eastern coastal plains, the western coastal plains are much broader.
Select the correct code from the following
a) 1 and 2
b) 2 and 3
c) 1 and 3
d) None of the above
Q.12) Solution (d)
Doubt- Sir kindly check ques no. 12 of day 3 that western coast is submerging n eastern
coast is emerging thats why western coast have several ports n harbours like Kandla,
Mazagaon, Nhava sheva etc. Page no. 18 India: Physical Environment (NCERT)
Explanation- All are incorrect but we asked for correct. So answer should be None of the
Above
Western coast is submerging while eastern coast is emerging.
Eastern coast is much broader than the western coast.
Large rivers make deltas on the eastern coast while rivers flowing towards west forms
estuaries.
Q.13) Consider the following statements about the Indian Thar desert
1. The underlying rock structure of desert is the extension of Peninsular plateau.
2. Fluvial erosion is common making mushroom rocks and shifting dunes.
3. Rivers flow in short streams and disappears showing inland drainage.

www.iasbaba.com

Geography- Subject wise compilation of 60 Day Plan- 2016

Which of the above statements are correct?


a) 1 and 2
b) 2 and 3
c) 1 and 3
d) All of the above
Q.13) Solution (c)
Aeolian (wind) erosion is dominant in this region.
Fluvial erosion is due to streams and rivers.
Q.14) Consider the following
1. The largest biomes are temperate grasslands, savannas and shrublands biomes
2. The region is colloquially referred as Stans
3. The region experiences very dry climatic conditions, and inadequate precipitation has
led to heavy dependence on the Syr Darya and Amu Darya for irrigation.
Identify the region based on given statements
a) Eastern Asia
b) Eastern Africa
c) Western Africa
d) Central Asia
Q.14) Solution (d)
Self-explanatory
Q.15) Consider the following statements with respect to LEGUMES
1. Legumes have an ability to fix nitrogen from the atmosphere
2. Legumes help in increasing soil fertility
3. Legumes are highly nutritive but also contain anti-nutrients that are harmful
Select the correct codes
a) 1 and 2
b) 2 and 3
c) 1, 2 and 3
d) Only 1
Q.15) Solution (c)
The legume family consists of plants that produce a pod with seeds inside. Common edible
legumes include lentils, peas, chickpeas, beans, soybeans and peanuts.

www.iasbaba.com

Geography- Subject wise compilation of 60 Day Plan- 2016


Cultivation of all the crops including cereals and regular vegetables will get benefitted if the
legume vegetables are grown once a season.
Apart from fixing atmospheric nitrogen, the legume vegetables also help in increasing the
organic content of the soil as they produce huge quantum of foliage that gets added into
the soil. In addition to this, they also enhance the water-holding capacity of soil, he points
out. They have a positive impact on human health too as these protein-rich legume
vegetables can help fight protein and energy-deficiency induced malnutrition.
They also contain so-called anti-nutrients like Phytic Acid, Lectins and Saponins which are
substances that can interfere with digestion and the absorption of other nutrients.
Doubt- Q19- its not legumes but rhizobia that fixes nitrogen from atmosphere and the
legumes that lack this symbiosis are incapable. hence st.1 incorrect.
Clarification- Did statement 1 mentions anything about any specific legume or did it say,
Legumes always?
Q.16) Emphasis to restore Jagannath temple at puri was in news recently. Consider the
following statements with respect to Puri Jagannath Temple
1. The idol of Jagannath temple at Puri is made of wood and is replaced once in twelve
years.
2. The temple was rebuilt in 12th century by Eastern Ganga dynasty King Anantavarman
Chodaganga Deva
3. The puri Jagannath temple constitutes one of the four Char Dham pilgrimages that a
Hindu is expected to visit in his lifetime.
Select the correct option
a) 1 only
b) 2 and 3 only
c) 1 and 2 only
d) All the above
Q.16) Solution (d)
https://en.wikipedia.org/wiki/Jagannath_Temple,_Puri
Q.17) Which of the following are correct about Karewas
1. Karewas are the lacustarine deposits found only in Kashmir valley and bhadarwa valley
of Jammu, in India.
2. It is suitable for growing one of the worlds best qualities of saffron.
3. The soil is peaty and contains a lot of carbon content.
Which of the above statements are correct?

www.iasbaba.com

Geography- Subject wise compilation of 60 Day Plan- 2016


a)
b)
c)
d)

1 and 2
2 and 3
1 and 3
All of the above

Q.17) Solution (d)


Karewas are lacustrine deposits [deposits in lake] in the Valley of Kashmir and in Bhadarwah
Valley of the Jammu Division. These are the flat topped mounds that border the Kashmir
Valley on all sides. They are characterized with fossils of mammals and at places by peat.
Q.18) Which of the following statements are correct about crop rotation?
1. Growing of different crops on a piece of land in a pre-planned succession is called crop
rotation.
2. The availability of moisture and irrigation facilities decide the choice of crops to be
grown.
3. Ideally staple grain crops are rotated with leguminous crops.
Select the correct code from above:
a) 1 and 2
b) 2 and 3
c) 1 and 3
d) All of the above
Q.18) Solution (d)
Usually grain crops like rice, wheat etc, needs a lot of nutrition. Repeated growing of these
crops rob the land from its nutrients. Hence it is ideal to rotate them with leguminous crops
so that their fertility can be regained through nitrogen fixation.
Q.19) Which of the following statements correctly explains the process of Ratooning?
a) It is a method of harvesting a crop which leaves the roots and the lower parts of the
plant uncut to give stubble crop.
b) It is a method of preparing field by burning the stubs of harvested crops in the field, to
provide nutrition to the soil.
c) It is method of using burrowing animals to make the field porous before harvesting of
crop.
d) None of the above.
Q.19) Solution (a)
Ratooning is a method of harvesting a crop which leaves the roots and the lower parts of
the plant uncut to give the ratoon or the stubble crop. One of the important methods of the
intensive cropping, allowing the stubbles of the original crop to strike again after harvesting

www.iasbaba.com

Geography- Subject wise compilation of 60 Day Plan- 2016


and to raise another crop.The main benefit of ratooning is that the crop matures earlier in
the season. Ratooning can also decrease the cost of preparing the field and planting.
This method cannot be used endlessly as the yield of the ratoon crop decreases after each
cycle. Ratooning is most often used with crops which are known to give a steady yield for
three years under most conditions.
Q.20) Arrange the following rock systems of our country as classified by the Geological
Survey of India from oldest to recent:
1. The Purana Rock System
2. The Dravidian Rock System
3. The Archaean Rock System
4. The Aryan Rock System
Select the correct code from the following:
a) 2, 3, 1, 4
b) 2, 3, 4, 1
c) 3, 1, 2, 4
d) 3, 1, 4, 2
Q.20) Solution (c)
Self explanatory
Q.21) Which amongst the following rivers has the largest drainage basin in India?
a) Indus
b) Brahmaputra
c) Krishna
d) Godavari
Q.21) Solution (a)
Doubt- Answer for q. 13 should be godavri since indus basin lies outside india. Could you
please give more detailed explanation and source links
Clarification
Amongst the given rivers Indus has the largest basin in India. It has the second largest basin
after river Ganga.
Ganga> Indua> Godavari> Krishna> Brahmaputra
http://india-wris.nrsc.gov.in/wrpinfo/index.php?title=Basins
Q.22) Consider the following statements:
1. Easterly jet stream over India is seasonal and exists only during summer.

www.iasbaba.com

Geography- Subject wise compilation of 60 Day Plan- 2016


2. Westerly jets are helpful in pushing western disturbances towards India, causing winter
rainfall.
3. Westerly jets are bifurcated by the Tibetan plateau during summers with one stream
flowing over northern India.
Which of the above statements are correct?
a) All of the above
b) 1 and 2
c) 2 and 3
d) 1 and 3
Q.22) Solution (b)
During winters, as the ITCZ shifts downwards, the subtropical high pressure belt also shifts.
With this shift, a downward shift in westerly jet stream is also seen. This jet stream when
strikes Pamir knot and tibetan plateau, it is bifurcated. One stream goes over the plateau
and one stream is pushed downwards, which flows over India.
Q.23) Consider the following statements:
1. The deviation of actual rainfall of a place from its average rainfall is called variability.
2. The highest variability is found at places where average rainfall is lowest.
3. Variability of rainfall is measured in percentage.
Which of the above statements are correct about variability of rainfall?
a) 1 only
b) 1 and 3
c) 2 and 3
d) All of the above
Q.23) Solution (d)
The actual rainfall of a place in a year deviates from its average rainfall by 10 to over 60
percent. The highest variability is found in areas which have less rainfall. For eg. In desert
area if average rainfall is only 20 cm, then even 10 cm increase in rain in a year would be
highly variable.
The variability of rainfall plays a significant role in agricultural and other economic activities.
Q.24) Which of the following statements are correct about black soils of India?
1. They have a volcanic origin.
2. They cover the largest reporting area of the country.
3. They are deficient in nitrogen, phosphorus and organic matter.
4. They have clayey texture and have high water holding capacity.

www.iasbaba.com

10

Geography- Subject wise compilation of 60 Day Plan- 2016


Select the correct code from the following:
a) 1 and 4
b) 1,2 and 3
c) 1,3 and 4
d) All of the above
Q.24) Solution (c)
Alluvial soils are most abundant covering 43.4% of reporting area. The second most
abundant soil is red soil with 18.5% area and then black soil with 15% area.
Q.25) Consider the following statements about the texture of soil:
1. The size of soil particles defines the texture of soil.
2. A soil having mixture of substantial amount of sand, silt and clay is called loam.
3. Soil texture decides the water holding capacity of the soil.
4. Finer the soil texture, the better it is for agriculture.
Which of the above statements are incorrect?
a) 1,2 and 3
b) 1 and 4
c) 4 only
d) None of the above
Q.25) Solution (c)
Medium sized particles are ideal for agriculture. (Silty loam)
Very fine particles have high water holding capacity and lacks aeration. Also they are hard to
work upon as they become sticky when wet and very hard when dry.
Q.26) Soil salinity has emerged a major problem affecting the fertility of soil in India.
Almost 2.5% of countrys reporting area is adversely affected by alkali formations. Which
of the following statements are correct about saline soils?
1. Salinity is naturally found in arid and semi-arid areas where rate of evaporation is higher
than rate of precipitation.
2. Saline soils develop in highly irrigated area as more salt comes up due to capillary action
of soil.
3. Salinity can be cured by adding gypsum to the soil.
Select the correct code from the following:
a) 1 and 3
b) 1 and 2
c) 2 and 3
d) All of the above

www.iasbaba.com

11

Geography- Subject wise compilation of 60 Day Plan- 2016

Q.26) Solution (d)


Natural salinity is found in desert regions like Rann of Kutch. These natural salt formations
are commercially utilized.
Q.27) Consider the following statements:
1. Kerala has no east flowing rivers.
2. Madhya Pradesh has no west flowing rivers.
Which of the above statements are correct?
a) 1 only
b) 2 only
c) Both 1 and 2
d) Neither 1 nor 2
Q.27) Solution (d)
There are three rivers rise in Kerala and flow eastwards, Kabini into Karnataka and the other
two into Tamil Nadu.
Kabani
Bhavani
Pambar
Q.28) Which one of the following rivers do not originate in India?
a) Ravi
b) Beas
c) Sutluj
d) Chenab
Q.28) Solution (c)
The origin of the Sutlej River is located close to Lake Rakhastal in Tibet in the vicinity of the
Kailash Mountain Range. The Sutlej River usually runs according to a west to southwest
itinerary moving into India via the Shipki La Pass located in Himachal Pradesh.
Q.29) Consider the following statements regarding equatorial climate:
1. Very heavy down pour, normally for a short duration in a year.
2. Convectional rainfall characterised by lightning and thunder.
3. There is a very high diurnal temperature range in equatorial regions.
Select the correct answer
a) 1 and 2 only
b) 2 only

www.iasbaba.com

12

Geography- Subject wise compilation of 60 Day Plan- 2016


c) 2 and 3 only
d) None of the above
Q.29) Solution (b)
Equatorial regions have very low diurnal temperature range and rainfall occur throughout
the year.
Q.30) The recent advent of yellow fever in some parts of Africa was in news recently.
Consider the following statements with respect to Yellow fever
1. The disease is transmitted by the bite of a Mosquito.
2. It is curable.
3. The disease is characterised by high fever and Jaundice.
Select the correct answer
a) 1 and 3 only
b) 2 only
c) 2 and 3 only
d) 1,2 and 3
Q.30) Solution (a)
Yellow fever is a serious, potentially deadly flu-like disease spread by mosquitoes.
Characterized by a high fever and jaundice, it is most prevalent in certain parts of Africa and
South America. The disease is not curable, but is preventable with the yellow fever vaccine.
Q.31) Recently ISRO test fired Prithvi 2 missile from a test range at Chandipur in Odisha.
Conisder the following statements with respect to Prithvi 2 missile
1. It is a surface to air missile
2. It has a strike range of 2000 km
3. It is the first missile developed under integrated missile development programme.
4. It has a single stage liquid fuelled propellant
Select the correct answer
a) 1 and 2 only
b) 2 and 3 only
c) 3 and 4 only
d) 1 and 3 only
Q.31) Solution (c)
Doubt- Dear babaji,,Prithivi-2 is test fired by DRDO as per Hindu not by ISRO...please clarify
it

www.iasbaba.com

13

Geography- Subject wise compilation of 60 Day Plan- 2016


Clarification- Yes it is a typo from our side. It is test fired by DRDO not ISRO
Q.32) PM referred to Pragjyotishpur at the inaugural address of south Asian games.
Which of the following city today refers to Pragjyotishpur in ancient India?
a) Dibrugarh
b) Thimpu
c) Dispur
d) Guwahathi
Q.32) Solution (d)
Pragjyotishpur in Ancient India refers to Guwahati in Modern India
Q.33) ARYA and Farmers first to revamp agricultural produce in India are proposed
initiatives by
a) Indian agricultural research institute
b) Ministry of agriculture in collaboration with ICRISAT
c) Indian council of agricultural research
d) Ministry of agriculture in collaboration with FAO
Q.33) Solution (c)
ARYA (Attracting and retaining youth in Agriculture), Mera Gaon Mera Gaurav and farmers
first initiatives by ICAR
Q.34) Which among the following are benefits of Atmosphere?
1) It protects from harmful radiation
2) It contains living gases
3) Acts as Green house, allows long-wave radiation and traps short-wave radiation
4) Acts as medium for fast air transport
5) Storehouse for water vapor and leads to precipitation
Choose the appropriate code:
a) 2, 3 and 5 only
b) 1, 2, 3 and 5 only
c) 1, 2, 4 and 5 only
d) All of the above
Q.34) Solution (c)
Only 3rd statement is wrong, because atmosphere acts as Green house by allowing
short-wave radiation (from Sun) and trapping long-wave terrestrial radiation (from
Earths surface)

www.iasbaba.com

14

Geography- Subject wise compilation of 60 Day Plan- 2016


Q.35) Arrange the following rays according to their wavelengths (from short-wavelength
to long)
1) X-rays
2) Gamma-rays
3) UV-rays
4) Visible rays
5) Infrared rays
Choose the appropriate code:
a) 2-1-3-5-4
b) 1-2-3-5-4
c) 1-2-3-4-5
d) 2-1-3-4-5
Q.35) Solution (d)
The types of electromagnetic radiation are listed below in fig.

(See: http://www.darvill.clara.net/emag/images/scale.jpg)
Q.36) Consider the following statements in regard to Westerlies:
1) Westerlies blow from the Sub-tropical high Pressure Belt to the Sub-polar low Pressure
Belt in the temperature latitudes between 30 N and 60 S latitudes
2) They are more constant and stronger in the Southern Hemisphere because there are no
large landmasses to interrupt them
3) It got the name Westerlies because they blow out of the west

www.iasbaba.com

15

Geography- Subject wise compilation of 60 Day Plan- 2016

Which of the statements given above is/are incorrect?


a) 1, 2 and 3 only
b) 1 and 2 only
c) 1 only
d) None
Q.36) Solution (c)

Westerlies blow from the Sub-tropical high Pressure Belt to the Sub-polar low Pressure
Belt in the temperature latitudes between 30 and 60, on either side of the equator
They are more constant and stronger in the Southern Hemisphere because there are no
large landmasses to interrupt them
In places they become so strong, these winds are known as Roaring Forties or the Brave
West winds and the Furious Fifties
The belts of the Westerlies move north and south following the Suns movement. These
are known as Westerlies because they blow out of the west.

Q.37) Which of the following are non-metallic minerals?


1) Gypsum
2) Quartz
3) Lead
4) Mica
Choose the appropriate code:
a) 3 and 4 only
b) 1, 2 and 4 only
c) 1, 3 and 4 only
d) All of the above
Q.37) Solution (b)
Lead is metallic mineral, where others are non-metallic
Q.38) Which among the following statements is/are true in regard to El-Nino?
1) El Nino is a narrow warm current which occasionally appears off the coast of Peru in
December
2) El Nino has the potential of increasing the surface water temperatures of the sea by
10C
3) El Nino refers to a cyclical warming of the Central and Eastern Equatorial Pacific that
frequently corresponds to a drought in India

www.iasbaba.com

16

Geography- Subject wise compilation of 60 Day Plan- 2016


Choose the appropriate code:
a) 2 and 3 only
b) 1 and 2 only
c) 1 and 3 only
d) All of the above
Q.38) Solution (d)
Self-explanatory All the statements are correct
Source: Chapter 5 Climate, D. R. Khullar
Doubts: Q no 5. "EL NINO IS A WARM CURRENT"??? Yeh kab se ho gaya????
I thought EL-NINO is a "PHENOMENON" of warming of central and eastern Pacific Ocean!!!
Clarification: Tsunami is generally thought by many and also called as an event or
phenomenon; however the geographical meaning of Tsunami is shallow water waves.
Q.39) Arrange the following Iron ores based upon their quality or pure iron content:
1) Limonite
2) Siderite
3) Magnetite
4) Haematite
Choose the appropriate code:
a) 1-2-3-4
b) 3-4-1-2
c) 3-4-2-1
d) 4-3-2-1
Q.39) Solution (b)
Magnetite: This is the best quality of iron ore and contains 72 per cent pure iron. It
possesses magnetic property and hence is called magnetite. It is found in Andhra
Pradesh, Jharkhand, Goa, Karnataka etc
Haematite: It contains 60 per cent to 70 per cent pure iron and is found in Andhra
Pradesh, Jharkhand, Orissa, Chhattisgarh, Goa etc
Limonite: It contains 40 per cent to 60 per cent pure iron. It is of yellow or light brown
colour
Siderite: It contains many impurities and has just 40-50 per cent pure iron. However, due
to presence of lime, it is self fluxing
Q.40) Consider the following statements in regard to Depressions:

www.iasbaba.com

17

Geography- Subject wise compilation of 60 Day Plan- 2016


1) Depressions are low pressure systems that form in the mid-latitude region (30-60 N/S)
when warm air meets cold air
2) When this occurs the warm air is forced above the colder air. It then cools and clouds
form which produce rain
3) Winds in depressions move in an clockwise direction in the northern hemisphere and
anti-clockwise in the southern hemisphere
4) Depressions are characterized by fronts
Which of the statements given above is/are correct?
a) 2, 3 and 4 only
b) 2 and 4 only
c) 1, 2 and 4 only
d) All of the above
Q.40) Solution (c)
In a depression (also referred to as a 'low'), air is rising. As it rises and cools, water
vapour condenses to form clouds and perhaps precipitation. Consequently, the weather
in a depression is often cloudy, wet and windy (with winds blowing in an anticlockwise
direction around the depression). There are usually frontal systems associated with
depressions.

Depressions are low pressure systems that form in the mid-latitude region (30-60 N/S)
when warm air meets cold air. When this occurs the warm air is forced above the colder
air. It then cools and clouds form which produce rain.

Winds in depressions move in an anti-clockwise direction in the northern hemisphere


and clockwise in the southern hemisphere (hence 3rd statement is wrong)
Source: Chapter 12 Atmospheric Pressure, Winds and Airmasses, Principles of Geography
Doubts:
It says "Depressions are low pressure systems that form in the mid-latitude region (30-60
N/S) when warm air meets cold air".
The statement is ambiguous. It should have mentioned Temperate or Tropical depressions
because Depressions also formed in tropical areas which further get enhanced to cyclone.
This statement makes it feel like depressions are associated with Temperate regions only
which is not correct.
Clarifications: There is nothing wrong with the statement. If the statement read Depressions are low pressure systems that form only in the mid-latitude region (30-60 N/S)
when warm air meets cold air, then the statement would be incorrect.
Moreover, globally, low-pressure systems or depressions are most frequently located over
the Tibetan Plateau and in the lee of the Rocky Mountains.
Q.41) Humidity of the air
a) Increases with the increase in atmospheric temperature

www.iasbaba.com

18

Geography- Subject wise compilation of 60 Day Plan- 2016


b) Decreases with the increase in atmospheric temperature
c) Is not affected by the change in atmospheric temperature
d) Does not show any consistent behaviour with the change in atmospheric temperature
Q.41) Solution (d)
Humidity means the amount of water vapour in the air. The temperature of the air
influences the amount of water vapor that can be bound to the molecules of air.
Water vapor capacity increases with temperature increase.
If the temperature is warm, the atmosphere has a greater capacity to hold water in its
vaporous state than if it were cold.
Case I: In Equatorial Region With increase in temperature, the amount of water vapour in
the air increases (i.e, humidity of the air increases with the increase in atmospheric
temperature)
Case II: Desert Region Despite high atmospheric temperature, humidity is low (i.e. amount
of water vapour in the air is less due to less water bodies or moisture availability in the
region)
Humiditywater vapor in the airis near zero in most deserts. We can say that it is not
affected much by the change in atmospheric temperature.
Therefore, option (d) Does not show any consistent behaviour with the change in
atmospheric temperature is the most appropriate answer, as water vapour acts as a prime
factor in humidity and not just temperature.
Q.42) Consider the following statements with regard to cyclones:
1) The centre of Tropical cyclone is characterized by extremely low pressure. There is no
temperature variation in their different parts.
2) Normally Tropical cyclones move from east to west under the influence of trade winds
and they mainly occur in summer.
3) Temperate Cyclones always travel from west to east and are produced only in winter.
4) Extratropical cyclones have cold air at their core, and derive their energy from the
release of potential energy when cold and warm air masses interact.
Which of the statements given above is/are correct?
a) 1, 2 and 4 only
b) 1 and 4 only
c) 2 and 4 only
d) 1, 2 and 3 only

www.iasbaba.com

19

Geography- Subject wise compilation of 60 Day Plan- 2016


Q.42) Solution (a)

The centre of Tropical cyclone is characterized by extremely low pressure.


Normally Tropical cyclones move from east to west under the influence of trade winds
and they mainly occur in summer.
Temperate Cyclones always travel from west to east and more cyclones are produced in
winter than in summer. (not only in winter)
Extra-tropical cyclones have cold air at their core, and derive their energy from the
release of potential energy when cold and warm air masses interact. These storms
always have one or more fronts connected to them, and can occur over land or ocean.

Doubts: Q first statement. wrong! Attaching a link of Britannica. Please read carefully the
second paragraph under "The Eye". Link: http://www.britannica.com/science/tropicalcyclone

Clarification: Statement 1 was modified (There is no temperature variation in their different


parts This part was removed)
Read same link properly - http://www.britannica.com/science/tropical-cyclone
1st paragraph under The Eye:
A characteristic feature of tropical cyclones is the eye, a central region of clear skies, warm
temperatures, and low atmospheric pressure. Typically, atmospheric pressure at the surface
of Earth is about 1,000 millibars. At the centre of a tropical cyclone, however, it is typically
around 960 millibars, and in a very intense super typhoon of the western Pacific it may be
as low as 880 millibars. In addition to low pressure at the centre, there is also a rapid
variation of pressure across the storm, with most of the variation occurring near the centre.
Q.43) Which among the following statements is/are not true with regard to monsoons?
1) Southwest monsoon brings rain during summer whereas Northeast monsoon brings rain
during winter
2) Direction of Northeast monsoon is land to sea so it doesnt contains moisture and brings
dryness and coldness after blowing through Bay of Bengal and brings rainfall only in
Tamil Nadu
3) During Southwest monsoon, Indian Subcontinent has high pressure and the direction of
air movement is from Australia to Indian subcontinent
Choose the appropriate code:
a) 2 and 3 only
b) 1 and 2 only
c) 2 only
d) 3 only
Q.43) Solution (d)
www.iasbaba.com

20

Geography- Subject wise compilation of 60 Day Plan- 2016

During Southwest monsoon, Indian Subcontinent has low pressure and the direction of
air movement is from Australia(high) to Indian subcontinent(low)
Northeast monsoon brings rain not only in Tamil Nadu but also other regions like Andhra
Pradesh, Puducherry

Clarification: This question has been corrected for a factual error. Please refer to the
modified second statement in the question above.
Q.44) Consider the following minerals
1. Limestone
2. Natural Gas
3. Gypsum
4. Uranium
5. Iron ore
6. Thorium
The minerals included in the wholesale price index are?
a) 1, 2, 3 and 5
b) 2, 3, 4, 5 and 6
c) 1, 3 and 5
d) 1, 3, 4, 5 and 6
Q.44) Solution (c)
The minerals included in the wholesale price index are bauxite, chromite, iron ore, copper
ore, zinc concentrate, manganese ore, barytes, dolomite, fireclay, gypsum, kaolin,
limestone, magnesite, phosphorite, graphite, steatite and sillamanite.
Source- Ministry of Mines- Annual Report
http://www.mines.nic.in/writereaddata/UploadFile/Mines_AR_2015-16_English.pdf
Q.45) The number of mines which reported mineral production in India has decreased
over the period of time. Consider the following
www.iasbaba.com

21

Geography- Subject wise compilation of 60 Day Plan- 2016


1.
2.
3.
4.
5.

Madhya Pradesh
Gujrat
Jharkhand
Tamil Nadu
Andra Pradesh

Arrange in decreasing order of number of mines present in the state?


a) 1-2-3-4-5
b) 2-1-3-5-4
c) 4-3-1-2-5
d) 1-3-2-4-5
Q.45) Solution (d)
Indian mining industry is characterized by a large number of small operational mines. The
number of mines which reported mineral production (excluding minor minerals, petroleum
(crude), natural gas and atomic minerals) in India was 1,878 in 2015-16 as against 3,524 in
the previous year. Out of 1878 reporting mines, 245 were located in Madhya Pradesh
followed by Jharkhand (208), Gujarat (192), Tamil Nadu (185), Andhra Pradesh (169), Odisha
(158), Chhattisgarh (154), Maharashtra (121), Karnataka (115), West Bengal (101), Goa (75)
and Rajasthan (66).
Q.46) Consider the following
1. A group of volcanic islands
2. Group of coral islands
3. A group of islands that lie in the centre of the oceans
4. A group of islands that lie in close proximity
Which of the above statements does not belong to the category of Archipelago?
a) 1, 2 and 3 only
b) 2 and 3 only
c) 1,2 and 4 only
d) All
Q.46) Solution (a)
A group of islands that lie in close proximity is known as archipelago
Q.47) Consider the following
1. Bhangar
2. Terai
3. Bhabar
4. Khadar

www.iasbaba.com

22

Geography- Subject wise compilation of 60 Day Plan- 2016

As one travels from the Gangetic plain towards Shiwaliks in the north, the correct sequence
of regions he/she will cross is?
a) 1234
b) 1243
c) 4321
d) 4123
Q.47) Solution (d)
Source Ncert 11th Physical environment
Similar explanation can also be found in NCERT also
The Bhabar belt is adjacent to the foothills of the Himalayas and consists of boulders and
pebbles which have been carried down by the river streams. As the porosity of this belt is
very high, the streams flow underground. The bhabar is generally narrow about 715 km
wide.
The Terai belt lies next to the Bhabar region and is composed of newer alluvium. The
underground streams reappear in this region. The region is excessively moist and thickly
forested. It also receives heavy rainfall throughout the year and is populated with a variety
of wildlife.
The Bangar belt consists of older alluvium and forms the alluvial terrace of the flood
plains. In the Gangetic plains, it has a low upland covered by laterite deposits.
The Khadir belt lies in lowland areas after the Bangar belt. It is made up of fresh newer
alluvium which is deposited by the rivers flowing down the plain.
Q.48) Salma Dam is in news. Salma Dam is in which of the following countries
a) Afghanistan
b) Iran
c) Sri Lanka
d) Bhutan
Q.48) Solution (a)
Salma dam is present in Afghanistan. PM Modi will inaugurate the dam as it is constructed
by India. The dam is expected to provide irrigation facilities for nearly 40 thousand hectars
of land.
Q.49) A great variety of soils is found over the globe. Which among the following are
causes for this variation?
1) Nature of rocks
2) Physical characteristics of land
3) Climate
4) Vegetation cover

www.iasbaba.com

23

Geography- Subject wise compilation of 60 Day Plan- 2016

Choose the appropriate code:


a) 1 and 3 only
b) 1, 2 and 3 only
c) 1, 3 and 4 only
d) All of the above
Q.49) Solution (d)
Explanation:
Self-explanatory, all the given factors are responsible for the variations of soils
Source: Chapter 7 Soils, Principles of Geography (Old NCERT)
Q.50) Which among the following statements is/are true in regard to soils?
1) Soil is not only a mixture of solid and liquid substances but also gaseous substances
2) Soil is a non-renewable resource
3) The rate of removal of fine particles from the surface (due to running water and wind) is
the same as the rate of addition of particles to the soil layer.
4) Soils with high lime content are called acid and those with low lime content are called
alkaline
Choose the appropriate code:
a) 3 and 4 only
b) 1, 2 and 3 only
c) 1, 3 and 4 only
d) All of the above
Q.50) Solution (b)
Explanation:
Only 4th Statement is wrong - Soils with low lime content are called acid and those with
high lime content are called alkaline
Source: Chapter 7 Soils, Principles of Geography (Old NCERT)
Doubts: @IASbaba- soil is a renewable resource as per geography ncert
Clarification: Refer Chapter 7 Soils Principles of Geography (Old NCERT)
Excerpt from that chapter Even in flatlands like Indias Ganga Plains, the soils get
exhausted and less productive every year if right methods of cultivation are not employed
to keep their productivity. It is given to man to adopt all measures to conserve the soil
resource. Once destroyed or degraded, the loss in terms of agriculture and plant cover is
more or less permanent. It is in this sense that soil is truly a non-renewable resource.

www.iasbaba.com

24

Geography- Subject wise compilation of 60 Day Plan- 2016


Refer
FAO
website:
details/en/c/278954/)

(http://www.fao.org/resources/infographics/infographics-

Soil is a non-renewable resource. Its preservation is essential for food security and our
sustainable future. Soil is a finite resource, meaning its loss and degradation is not
recoverable within a human lifespan.
Q.51) Consider the following statements:
1) Laterite soils are formed due to the heat combined with the rate of alternation between
wet and dry intervals of monsoon climate
2) Hot summer and low rainfall develops black soil, irrespective of the parent rock
3) In tropical/equatorial region since biodiversity/vegetation is thick, humus content is also
very high
Which of the statements given above is/are correct?
a) 2 and 3 only
b) 1 and 2 only
c) 1 and 3 only
d) All of the above
Q.51) Solution (b)
Explanation:
Climate of an area is such a major factor that over a long period, it tends to reduce
differences caused by the parent materials. That is why two different parent material
may develop the same soil in one type of climatic regime.
Likewise, the same parent material may produce two different types of soil in two types
of climates.
Statement 1 is correct self explanatory; Laterite has been derived from the Latin word
Later which means brick. The laterite soils develop in areas with high temperature and
high rainfall.
Statement 2 is also correct and above explanation is the reason why black soil are
found in some districts of Tamil Nadu, irrespective of the parent rock
Statement 3 is wrong. Tropical/equatorial region consists of humid climate, therefore
bacterial activity is intense. Consumption of humus is so much that soils are left poor in
it.

In cold climate (temperate region), bacterial activity is limited and the soils are rich in
humus.
Source: Chapter 6 Soils, India Physical Environment and Chapter 7 Soils, Principles of
Geography (Old NCERT)
Q.52) Which among the following statements is/are not true?

www.iasbaba.com

25

Geography- Subject wise compilation of 60 Day Plan- 2016


1) Pedocals are soils that are found in humid climates extending from high-latitude
coniferous forests, mid latitude deciduous forest lands to low-climate tropical forests
and grasslands
2) Pedalfers are soils that are found in arid, semi-arid and the sub-humid zones in the
world
3) Pedocals are very rich in calcium and mineral salts
Choose the appropriate code:
a) 1 and 2 only
b) 2 and 3 only
c) 3 only
d) None of the above
Q.52) Solution (a)
Explanation:
Pedalfers are soils that are found in humid climates extending from high-latitude
coniferous forests, mid latitude deciduous forest lands to low-climate tropical forests
and grasslands
Pedocals are soils that are found in arid, semi-arid and the sub-humid zones in the world
Pedocals are very rich in calcium and mineral salts
Source: Chapter 7 Soils, Principles of Geography (Old NCERT)
Q.53) Consider the following statements:
1) These soils are called black earths
2) They are rich in humus, rich in calcium, unleached and have a crumby or nut-like
structure
3) These are fertile soils requiring little fertilizing even when used year after year.
4) The Steppes of Ukraine, the central part of USA, central Africa, South America and
Australia have these type of soils
Select the appropriate soil which the statements are referring to:
a) Chernozems
b) Prairie soils
c) Grey brown Podsolic soil
d) Red and Yellow Pedalfers
Q.53) Solution (a)
Explanation:
Self-explanatory
Source: Chapter 7 Soils, Principles of Geography (Old NCERT)

www.iasbaba.com

26

Geography- Subject wise compilation of 60 Day Plan- 2016

Q.54) Which of the following are true about Desert soils?


1) They are grey in colour in temperate region and red in hot deserts of the tropics
2) The soils are unleached and alkaline
3) These soils are very much lacking in humus because of little vegetation found
Choose the appropriate code:
a) 2 and 3 only
b) 1 and 2 only
c) 1 and 3 only
d) All of the above
Q.54) Solution (d)
Explanation:
Desert soils are grey in colour in temperate region and red in hot deserts of the tropics
The soils are unleached and alkaline but very much lacking in humus because of little
vegetation found
Source: Chapter 7 Soils, Principles of Geography (Old NCERT)
Q.55) Consider the following statements with regard to Usar and Reh:
1) Usar is a term collectively applied to all kinds of saline and alkaline soils in the plains of
north India, particularly in Uttar Pradesh
2) Reh is a white, grayish or ash-coloured salt that are found in low-lying plain areas in dry
periods
Which of the statements given above is/are incorrect?
a) 1 only
b) 2 only
c) Both 1 and 2
d) Neither 1 nor 2
Q.55) Solution (d)
Explanation:
Both the statements are correct and self explanatory
Source: Chapter 6 Soils, India Physical Environment and Chapter 7 Soils, Principles of
Geography (Old NCERT)
Q.56) Consider the following statements:

www.iasbaba.com

27

Geography- Subject wise compilation of 60 Day Plan- 2016


1) Alluvial soils are rich in potash and phosphorous
2) The colour of the alluvial soils varies from the light grey to ash grey
3) Sikkim has only Mountain soil and no presence of Alluvial/Terai soil
Which of the statements given above is/are correct?
a) 2 only
b) 1 and 2 only
c) 1 and 3 only
d) All of the above
Q.56) Solution (a)
Explanation:
Alluvial soils are rich in potash but poor in phosphorous
The colour of the alluvial soils varies from the light grey to ash grey
Sikkim has both Mountain soil and Terai soil (which is alluvial)
Source: Chapter 6 Soils, India Physical Environment and Orient BlackSwan Atlas (Thematic
map Soils)
Q.57) Which among the following statements is/are correct with regard to Zonal,
Intrazonal and Azonal Soils?
1) Soil occurring over wide areas on well-drained land which have been there long enough
for the climate and organisms to have expressed their full influence is called Zonal Soils
2) Soils affected by some local conditions not involving climate or vegetation, such as poor
drainage or those where parent material exerts a strong influence is Intrazonal soils
3) Immature soils without well developed soil characteristics are called Azonal Soils
Choose the appropriate code:
a) 2 and 3 only
b) 1 and 2 only
c) 1 and 3 only
d) All of the above
Q.57) Solution (d)
Explanation:
Zonal Soils:
The zonal soils are formed on normal sites from ordinary siliceous rocks and show clearly
the impress of climate and vegetation. In short, these are formed under conditions of
good soil drainage through the prolonged action of climate and vegetation, e.g. chestnut
soils. This kind of soils have following types 1. Tundra Soils

www.iasbaba.com

28

Geography- Subject wise compilation of 60 Day Plan- 2016


2.
3.
4.
5.
6.
7.

Podzols
Brown Forest Soils
Laterite Soils/Latosols/Ferralsols
Chernozem/Prairie/Steppe
Grumusol/Reddish Brown Soils
Desert (Seirozems and Red Desert) Soils

Intrazonal Soils:
The intrazonal soils include the soils from less common parent materials and those
influenced by high ground water or under conditions of very poor drainage (such as in
bogs, flood-plain meadows, or in the playa lake basins of the deserts) or upon limestone.
Depending on the role played by water, presence of calcium in the parent material and
the location, intra-zonal soils may be 1) Hydromorphic
2) Calcimorphic
3) Halomorphic
Azonal Soils:
The azonal soils are youthful, owing to recent renewal by sedimentation or erosion. This
have no well-developed profile characteristics. These soils are common where the
parent material is being continuously eroded and deposited, e.g. alluvial Soils (newer or
younger Khadar and older Bhangar soils) or lithosols (those at high altitudes on resistant
parent material). These soils have poorly developed horizons due to three reasons:
1) Lack of Time For instance, in new flood plains alluvium is being continually eroded and
deposited.
2) Parent Material Azonal soils like regosols result from loose sand and loess.
3) Geomorphology Lithosols result on steep slopes where soil is eroded as soon as it is
deposited.
Q.58) Consider the following characteristics with regard to Red soils:
1) Red soils are the largest soil group in India
2) Most of the red soils have come into existence due to weathering of ancient crystalline
and metamorphic rocks
3) They are usually poor growing soils, low in nutrients and humus and difficult to cultivate
because of its low water holding capacity.
4) Red and Yellow soil is abundantly found in southern belt i.e., Tamil Nadu and Karnataka
Choose the appropriate code:
a) 3 and 4 only
b) 1, 2 and 3 only
c) 1, 3 and 4 only

www.iasbaba.com

29

Geography- Subject wise compilation of 60 Day Plan- 2016


d) All of the above
Q.58) Solution (b)
Explanation:
Red soils are the largest soil group in India. Most of the red soils have come into
existence due to weathering of ancient crystalline and metamorphic rocks
They are usually poor growing soils, low in nutrients and humus and difficult to cultivate
because of its low water holding capacity.
Red and Yellow soil is not found in southern belt (i.e., Tamil Nadu and Karnataka) Refer
Orient BlackSwan Atlas (Thematic map Soils)
Due to presence of haematite and limonite its colour ranges from red to yellow, usually
found on Jharkhand, Chattisgarh and Orissa
Source: Chapter 7 Soils, D. R. Khullar and Refer Orient BlackSwan Atlas (Thematic map
Soils)
Doubt 1: @IASbaba Alluvial Soil is the largest soil group in India
Clarification: There are two aspects in regard to this statement (1)
As we all know, Alluvial soils are by far the largest and the most important soil group of
India.
Alluvial soils are scattered throughout the country and is the most widespread category.
These soils cover 40% of the entire land area in India. This soil covers an area of 15 lakh sq
km in India and mainly contributes in the development of agriculture. They form around the
lower courses of most rivers around the country and particularly all over the Indo-Gangetic
Plain. The entire Northern Plains consists of Alluvial soils. These soils are also extended into
the plains of Gujarat and are also found in the eastern coastal plains as well.
Note: But after thorough research and studying from many sources, we derived at other
aspect. (Explained below)
Red soils along with its minor groups form the largest soil group of India. The main parent
rocks are crystalline and metamorphic rocks like acid granites, gneisses and quartzites.
Red soil is the largest omnibus group. They are mainly developed over Archaean granite,
gneiss and other crystalline rocks, the sedimentaries of the Cuddapah and Vindhayan basins
and mixed Dharwarian group of rocks. Ordinarily the surface soils are red while the horizon
below gets yellowish colour.
Red Soil types in general:
(a) Red Loam Soil
(b) Sandy Red Soil
The Indian Council of Agricultural Research (ICAR) has divided red soils into four categories(a) red soils, (b) red gravelly soils, (c) red and yellow soils, and (d) mixed red and black soils.

www.iasbaba.com

30

Geography- Subject wise compilation of 60 Day Plan- 2016

A glimpse over the country's map will make one scrutinise that the north-western portions
of the peninsular block is covered by black soil while the rest south-eastern half is covered
by red soil of various shades of yellow and red. Red soils have matured on older crystalline
rocks, under deep and rational rainfall conditions. They basically surround the whole black
soil region on all sides, and cover the eastern part of the peninsula, comprising Chota
Nagpur Plateau, Orissa, eastern regions of Madhya Pradesh, Telangana, Tamil Nadu plateau
and Nilgiri Hills.
Doubt 2: Red and Yellow soil is abundantly found in southern belt i.e., Tamil Nadu and
Karnataka. But babaji your answer says it is incorrect. Check the figure in NCERT not orient
blackswan Atlas.
Clarification: Again same problem we faced. There were good number of sources which
showed Red and Yellow soils presence in Southern belt (TN and Karnataka) and also there
were same amount of sources which showed contrasting to it.
Links:
Red and yellow soils in southern belt (esp. in TN and Karnataka)

www.iasbaba.com

31

Geography- Subject wise compilation of 60 Day Plan- 2016


Red and yellow soils not in southern belt (esp. in TN and Karnataka)
http://www.mapsofindia.com/maps/india/india-map-of-soils.jpg
http://www.indmaps.com/thematic-map/images/india-soil.gif
http://www.indiamapatlas.com/thematic-map/images/soils-map-of-india.gif
http://envis.mse.ac.in/maps/nature/india-map-of-soils.jpg
Red and Yellow Soil (from NCERT India Physical Environment) page 71, Chapter Soils
Red soil develops on crystalline igneous rocks in areas of low rainfall in the eastern and
southern part of the Deccan Plateau. Along the piedmont zone of the Western Ghat, long
stretch of area is occupied by red loamy soil. Yellow and red soils are also found in parts of
Odisha and Chattisgarh and in the southern parts of the middle Ganga plain. The soil
develops a reddish colour due to a wide diffusion of iron in crystalline and metamorphic
rocks. It looks yellow when it occurs in a hydrated form.
*NOTE: It is in your discretionary to choose which ever answer given in explanation above.
But let us consider the answer as statement 2 and 3 is incorrect. (following NCERT)
So question should be read as Q.) Consider the following characteristics with regard to Red soils:
1. Red soils are the largest soil group in India
2. Most of the red soils have come into existence due to weathering of
ancient crystalline and metamorphic rocks
3. They are usually poor growing soils, low in nutrients and humus and
difficult to cultivate because of its low water holding capacity.
4. Red and Yellow soil is abundantly found in southern belt i.e., Tamil
Nadu and Karnataka
Choose the appropriate code:
a) 3 and 4 only
b) 2 and 3 only
c) 1, 3 and 4 only
d) All of the above
Q.) Solution (b)
Q.59) There is a lot of fuss created about UK exiting the EU (European Union). Consider
the following statements with respect to European Union
1. EU is economic political union comprising of 28 countries.
2. The headquarters of EU is in Brussels, Belgium
3. Among the EU cities Paris is the largest city
Select the correct answer using the codes given below
a) 1 and 2 only
b) 2 and 3 only

www.iasbaba.com

32

Geography- Subject wise compilation of 60 Day Plan- 2016


c) 1 and 3 only
d) All
Q.59) Solution (a)
Among the EU London is the largest city
Q.60) Consider the following statements regarding a particular soil
1. Found in the summits of Western Ghats, Eastern Ghats, Vindhyas, Sathpuras and Malwa
plateau.
2. Subjected to intense leaching and thus not fit for agriculture unless manures and
fertilizers are used.
3. It is the end product of weathering process and thus it is indefinitely durable.
Which is the soil that is spoken about in the above statements?
a) Forest soil
b) Mountain Soil
c) Laterite soil
d) Red soil
Q.60) Solution (c)
Although first two statements ie 1 and 2 do comply with mountain soil and forest soil, the
third statement is unique for laterite soil.
Q.61) Consider the statements regarding red and yellow soils.
5. Soil looks yellowish when iron diffused in the soil gets hydrated.
6. Red and yellow soils are mainly found in the deltaic regions especially in the Eastern
Indian coast.
7. Less fertile coarser soils are found in the uplands while fertile fine soils occur in the low
lying regions.
Choose the correct code
a) 1 only
b) 1 and 2 only
c) 1 and 3 only
d) 2 and 3 only
Q.61) Solution (c)
This type of soils get red colour when iron is diffused in the crystalline and metamorphic
rocks. The same soil turns yellow when the iron diffused gets turned into hydrated form.
These soils occur generally in the drier regions of Eastern and Southern India and not in the
deltaic regions.

www.iasbaba.com

33

Geography- Subject wise compilation of 60 Day Plan- 2016

Q.62) Black soils are known for its fertility with little or no evidence of exhaustion. This is
because
1. Black soils with equal proportion of clay, gravel and coarse sand has high moisture
retention capacity
2. Cracks developed during dry season allow oxygenation of the soils.
3. Black soil has low level of kankar thus supports wide variety of crops.
Choose the incorrect reason from the codes below
a) 1 Only
b) 2 and 3 only
c) 1 and 3 only
d) All the above
Q.62) Solution (c)
Black soil is mainly composed of clay, thus it has high moisture retention capacity . Black
soils are in fact rich in kankar nodules which add to the fertility of the soil.
Q.63) Arrange the following countries in decreasing order of Average Cereal Yields
1. China
2. USA
3. India
4. Bangladesh
5. Brazil
6. Canada
Select the correct code
a) 2-1-3-5-6-4
b) 1-2-5-6-3-4
c) 2-1-5-4-6-3
d) 2-1-5-3-6-4
Q.63) Solution (c)
Economy Survey - Prices, Agriculture and Food Management, Fig 5.11
Q.64) Ragi, a millet is currently encouraged to include in the diet to supplement nutrients
to counter the malnutrition that is haunting the country. Consider the following
statements.
1. Ragi is rich in iron, calcium, roughage and is mainly grown in dry regions.
2. Among minor producers Sikkim, Arunachal Pradesh, Uttarakhand are included.

www.iasbaba.com

34

Geography- Subject wise compilation of 60 Day Plan- 2016


3. It is grown as a major crop in Bengaluru-Mysuru table land from where the countrys
majority of Ragi production comes from.
Choose the correct statements from the code below.
a) 1 and 2 only
b) 2 and 3 only
c) 1 and 3 only
d) All the above
Q.64) Solution (d)
Among minor producers even Jharkhand, Jammu Kashmir are also involved apart from the
states mentioned in the 2nd statement.
Ragi production and consumption is highest in Bengaluru-Mysuru table region.
Q.64) Tea best grows in Assam, Darjeeling and Jalpaiguri districts of West Bengal. What
is/are the uniqueness of this region which suits this crop?
1. It has warm and humid climate.
2. Showers are mainly concentrated in 4-5 months ie June to October in any year.
3. Deep, fertile and well-drained soil.
4. Rugged topography of the relief near the Himalayan foot hills.
Choose the correct code.
a) 1, 2 and 3 only
b) 1 and 3 only
c) 1, 3 and 4 only
d) All the above
Q.65) Solution (b)
Showers are almost evenly distributed throughout the year with very low dry spell.
Well drained gentle sloped soils are required and rugged topography is not suitable for tea
plantations.
Q.66) Sugarcane grown in subtropical region has lower productivity than that is grown in
tropical regions.
Consider the following statements about subtropical region i.e Northern India which has
low productivity in sugarcane production.
1. Frost formations
2. Floods and water logging
3. High diurnal temperature
4. Low humidity

www.iasbaba.com

35

Geography- Subject wise compilation of 60 Day Plan- 2016


Choose the correct reason/s from the codes below.
a) 1 and 2 only
b) 2 and 3 only
c) 3 and 4 only
d) 1, 2 and 3 only
Q.66) Solution (a)
First two statements are self-explanatory.
A higher diurnal temperature and low humidity are required for a better productivity. Thus
these two factors are not reducing the productivity.
Q.67) Consider the following statements:
1. A short duration growing season between Rabi and Kharif is called Zaid.
2. Zaid crops require warm dry weather for major growth period and longer day length for
flowering.
3. Major crops are seasonal fruits and vegetables like watermelon, musk melon, cucumber
etc.
4. Paddy seeds are sown in this season.
Which of the above statements are correct?
a) 1,2 and 3
b) 2,3 and 4
c) 1 and 3
d) All of the above
Q.67) Solution (d)
In the Indian sub-continent, the crops grown on irrigated lands which do not have to wait
for monsoons, in the short duration between Rabi and Kharif crop season, mainly from
March to June, are called Zaid crops (also written as Zayad crops). These crops are grown
mainly in the summer season during a period called the "Zayad crop season." They require
warm dry weather for major growth period and longer day length for flowering. The main
produce is seasonal fruits and vegetables.
Rice is a Kharif crop but its seeds are sown in Zaid season. In Zaid season the seeds grow into
saplings. These saplings are plucked and then transplanted in paddy field manually in Kharif
season.
Q.68) Consider the following statements regarding the milk production in India:
1. India is the Worlds leading milk producer.
2. The growth of Indias milk production in year 2015-16 is higher than the average Worlds
growth of milk production.

www.iasbaba.com

36

Geography- Subject wise compilation of 60 Day Plan- 2016


3. India has amongst the most productive breeds of milch animals.
Which of the above statements are correct?
a) 1 and 2
b) 2 and 3
c) 1 and 3
d) All of the above
Q.68) Solution (a)
According to economic survey 2015-16, India recorded a growth of 6.26%. Whereas, FAO
has reported an average worldwide growth of 3.1%.
Indian Milch Breeds are amongst the least productive of the world. Programs have been
made to cross breed Indian breeds with foreign breeds to increase their productivity.
Q.69) Sugar is one of the basic human necessities across the world. Sugarcane plantations
became a major driver of large and forced human migrations in 19th and early 20th
century. Consider the following statements regarding sugarcane and sugar production:
1. It was in India that farmers discovered and began the farming of sugarcane.
2. The process to produce sugar crystals was invented in India.
3. European merchants started trading in sugar, which was a luxury and expensive spice in
Europe.
4. Europeans from India took sugarcane to Europe and it became an important crop to be
grown in England and France.
Which of the above statements are correct?
a) 1,2 and 3
b) 2,3 and 4
c) 1,3 and 4
d) All of the above
Q.69) Solution (a)
Europeans started growing sugarcane in there colonies in tropical area. Climate of Europe is
not suitable for the growth of sugarcane.
Over 2500 years ago, Indian farmers had discovered and begun farming many spices and
sugarcane. It was in India, between the sixth and fourth centuries BC, that the Persians,
followed by the Greeks, discovered the famous "reeds that produce honey without bees"
being grown. pronounced as saccharum. On their return journey, the Macedonian soldiers
carried the "honey bearing reeds," thus spreading sugar and sugarcane agriculture. People
in India had invented, by about 500 BC, the process to produce sugar crystals. In the local
language, these crystals were called khanda , which is the source of the word candy.

www.iasbaba.com

37

Geography- Subject wise compilation of 60 Day Plan- 2016


The 18th century, cultivation of sugarcane was largely confined to India. A few merchants
began to trade in sugar a luxury and an expensive spice in Europe until the 18th century.
Sugar became widely popular in 18th-century Europe, then graduated to becoming a human
necessity in the 19th century all over the world. This evolution of taste and demand for
sugar as an essential food ingredient unleashed major economic and social changes.
Sugarcane does not grow in cold, frost-prone climate; therefore, tropical and semitropical
colonies were sought. Sugarcane plantations, just like cotton farms, became a major driver
of large and forced human migrations in 19th century and early 20th century of people
from Africa and from India, both in millions influencing the ethnic mix, political conflicts
and cultural evolution of Caribbean, South American, Indian Ocean and Pacific Island
nations.
Q.70) Consider the following statements regarding Truck Farming:
1. The crops are grown at the back of truck and directly sold in market.
2. Commercial growing of vegetables and fruits is called truck farming.
Which of the above statements is/are correct?
a) 1 only
b) 2 only
c) Both 1 and 2
d) Neither 1 nor 2
Q.70) Solution (d)
Horticultural practice of growing one or more vegetable crops (not fruits) on a large scale
for shipment to distant markets is called truck farming. It got its name as perishable crops
had to be transported quickly to distant markets.
This question created a number of questions and it need to be finally put to end.
IASbaba- Std XII NCERT "Fundamentals of Human Geography" Chap-5 Primary Activities,
Page 40. It states that "The regions where farmers specialize in vegetables only, the
farming is known as truck farming". So the answer should be D instead of B
Truck Farming basically is used for many types of perishable crops. In fact it has got nothing
to do with crop but perishability. Someone correctly explained this point in the comments,
that since they are taken to the market quickly by truck, this is called truck farming.
Mostly vegetables and some perishable fruits have to be taken to the market quickly as they
are plucked, like we get leafy vegetables, tomato etc in urban areas. These vegetables are
plucked early in the morning and are quickly sent to the retail markets. This includes many
fruits also.
Someone has given the example of strawberry. All berries, figs, as well as flowers comes in
the category of truck farming.

www.iasbaba.com

38

Geography- Subject wise compilation of 60 Day Plan- 2016


In Australia, vegetables and fruits are grown in Tasmania. Every morning they are plucked
and rushed to mainland Markets in fast steamers. This is a brilliant example of Truck
Farming.
There are some mistakes in NCERTs, and this is one of them.
The correct answer will remain (b)
Q.71) European Union has extended ban on import of various food items causing a loss of
approximately $2 million to India. Consider the following statements regarding EU ban on
Indian items:
1. EU has extended its ban indefinitely on the import of Indian mangoes.
2. The ban was imposed as fruit flies were discovered in some of the consignments of
mango.
Which of the above statements are correct?
a) 1 only
b) 2 only
c) Both 1 and 2
d) Neither 1 nor 2
Q.71) Solution (b)
Ban was imposed on mangoes for 20 months in 2014 after the detection of fruit flies in
some of the consignments. However it was exempted last year.
The European Union (EU) has extended the ban on import of four vegetables bitter gourd,
eggplant, taro plant (arbi) and snake gourd from India by another year despite efforts
made by the country to put in place stringent quality control and packaging measures.
Q.72) According to the report of Food and Agriculture Organisation, India is the largest
producer of which of the following agricultural products in the world?
1. Fresh fruits
2. Jute
3. Pulses
4. Rice
5. Wheat
Select the code from the following:
a) All of the above
b) 1,2,3 and 5
c) 1,2 and 3
d) 1 and 3
Q.72) Solution (c)

www.iasbaba.com

39

Geography- Subject wise compilation of 60 Day Plan- 2016


The Statistics Office of the Food and Agriculture Organisation reported that, per final
numbers for 2009, India had grown to become the world's largest producer of the following
agricultural products:
Fresh Fruit
Lemons and limes
Buffalo milk, whole, fresh
Castor oil seeds
Sunflower seeds
Sorghum
Millet
Spices
Okra
Jute
Beeswax
Bananas
Mangoes, mangosteens, guavas
Pulses
Indigenous buffalo meat
Fruit, tropical
Ginger
Chick peas
Areca nuts
Other bastfibres
Pigeon peas
Papayas
Chillies and peppers, dry
Anise, badian, fennel, coriander
Goat milk, whole, fresh
Per final numbers for 2009, India is the world's second largest producer of the following
agricultural products:
Wheat
Rice
Fresh vegetables
Sugar cane
Groundnuts, with shell
Lentils
Garlic
Cauliflowers and broccoli
Peas, green
Sesame seed
www.iasbaba.com

40

Geography- Subject wise compilation of 60 Day Plan- 2016

Cashew nuts, with shell


Silk-worm cocoons, reelable
Cow milk, whole, fresh
Tea
Potatoes
Onions
Cotton lint
Cottonseed
Eggplants (aubergines)
Nutmeg, mace and cardamoms
Indigenous goat meat
Cabbages and other brassicas
Pumpkins, squash and gourds

Q.73) Consider the following:


Term
Meaning
1. Floriculture
Production and marketing of Floral crops
2. Olericulture
Production and marketing of vegetables
3. Viticulture
Production and marketing of grapes
4. Oenology
Study of wine
Which of the above are correctly matched?
a) 1 and 3
b) 2 and 4
c) 1,3 and 4
d) All of the above
Q.73) Solution (d)
Self-explanatory
Q.74) Between 1970 and 2011, the GDP share of agriculture has fallen from 43% to 16%.
Which of the following statements is the correct reason for this?
1. The agricultural production gradually decreased.
2. There was a large scale rural to urban migration.
3. The secondary and tertiary sectors grew very rapidly.
Select the code from below:
a) 1 and 2
b) 3 only
c) 2 and 3
d) All of the above
www.iasbaba.com

41

Geography- Subject wise compilation of 60 Day Plan- 2016

Q.74) Solution (b)


First statement is incorrect as the agricultural production has grown rapidly.
Second statement is correct but is not the correct reason for the fall of share of agriculture
in GDP.
As the share of manufacturing sector and tertiary sector increased, the share of agriculture
relatively decreased.
Q.75) Coconut is an important crop with almost all its parts used for some or the other
purposes. Coconut is widely grown in India and India is the third largest producer of the
world after Indonesia and Philippines. Consider the following statements regarding
coconut:
1. There are two basic varieties of coconut, the tall and the dwarf.
2. The plant requires a year round warm and humid climate.
3. Coarse sand soil is deemed perfect for coconut plantations.
4. Coconut trees need saline water to grow.
Which of the above statements are correct?
a) 1,2 and 3
b) 2,3 and 4
c) 1,2 and 4
d) All of the above
Q.75) Solution (a)
There are only two varieties of coconuts; the tall and the dwarf. Tall varieties have a lifespan of around 80-90 years and coconut trees of this variety can grow up to a height of 15
to 18 meters. Whereas short varieties have an average life span of around 40 to 50 years
and they can grow up to a height of 5 to 7 meters. The tall and short varieties also have their
own types which grow in separate parts of the country. Furthermore, there are hybrid
varieties of coconut trees aimed at improved fruit quality.
Large-scale coconut production has its own pre-requisites like apt weather conditions,
proper soil, and proximity to the sea. To begin with, a coarse sand soil is deemed perfect for
coconut plantation. The plant requires a year-round warm and humid climate, and hence, a
close proximity to the sea as well as the equator is required. Lots of sunshine with high
humidity levels are suitable for coconut production. Temperatures above 24C and an
average annual rainfall of around 150 cm a year is also crucial for large-scale coconut
plantation. Coconut trees are rarely seen in low humidity areas and the plant does not
survive for too long in cold conditions. In fact, freezing and frosty conditions prove to be
fatal for the plant. The tree does not bear fruit amidst unsuitable weather conditions. The
coconut tree is a member of a group of plants called halophytes, that are tolerant of salt

www.iasbaba.com

42

Geography- Subject wise compilation of 60 Day Plan- 2016


water, which would kill most land plants. Scientists have been trying to figure out how the
halophytes extract the desalinated water they need to survive in hopes they could transferit
to crop plants which cannot do this. Cracking this mystery would let us grow crops in arid
soils where salt is concentrated in the soil and leaches out into irrigation or rain water. For
that reason, they have a good understanding of the two tricks the halophytes use to
desalinate their drinks.
Halophytes have two ways of achieving their salt tolerance. First, their root membranes are
adept at excluding the Na atom of NaCl molecules from their roots. This lets them take in
H2O and other needed minerals but block out the salt. In addition, some halophytes can
route salt to shoots having special compartments within their cells, called vacuoles which
are large storage vats where NaCl can be cordoned off and excluded from the areas on the
plant cell where it would do harm.
Q.76) Indian seeds programme largely adheres to limited generation system for seed
multiplication. The system recognises three kinds of generation, namely breeder,
foundation and certified seeds. Consider the following statements:
1. Breeder seed is the basic seed which is genetically pure and obtained from a healthy
plant.
2. Offsprings of the Breeder seed which can be clearly traced to Breeder seed are called
Foundation Seeds.
3. Certified seed is the last stage, which actually reaches to a farmer. Certified seed is the
progeny of foundation seed and must meet the standards of seed certification
prescribed in the Indian Minimum Seeds Certification Standards, 1988.
Which of the above statements are correct?
a) 1 and 2
b) 2 and 3
c) 1 and 3
d) All of the above
Q.76) Solution (b)
The Indian seed industry had exhibited impressive growth in the past and is expected to
provide further potential for growth in agricultural production: The role of seed industry is
not only to produce adequate quantity of quality seeds but also to achieve varietal diversity
to suit various agro-climatic zones of the country.
The policy statements are designed towards making available to the Indian farmer,
adequate quantities of seed of superior quality at the appropriate time and place and at an
affordable price so as to meet the countrys food and nutritional security goals.

www.iasbaba.com

43

Geography- Subject wise compilation of 60 Day Plan- 2016


Indian seeds programme largely adheres to limited generation system for seed
multiplication. The system recognises three kinds of generation, namely breeder,
foundation and certified seeds.
On this basis, there are five different categories viz. Nucleus Seeds, Breeders Seeds,
Foundation Seeds, Registered Seeds and finally certified seeds. The Offspring of breeder
seeds is foundation seeds to registered seeds to certified seeds.
Further, each of the breeders, foundation, registered and certified seeds are certified and
labelled with a different colour tag as per Section 5 of the Seeds Act, 1966. Thus, they are
also called as Labelled Seed. The Breeder seeds have golden yellow tag, Foundation seeds
have white tag, registered seeds have opal blue tag and certified seeds have green tag.
The process of development of certified seeds begins from its distant parent called
Nucleus Seed. The nucleus seed is a genetically pure seed without any impurity. They are
obtained from a handful of healthy plants growing in a plot and then grown strictly in
isolation.
A breeders seed is an offspring of nucleus seed. A breeder is a person (qualified plant
breeder) or organization who raises plant primary for breeding purpose. While nucleus
seed is genetically pure; the Breeder seeds are produced by mutual multiplication of
three different lines which are denoted by A line, B line and R line. A line is female, while
B and R line are male sterile. The seeds from off springs with best and desired quality are
selected and certified as Breeder seeds. Such seeds are protected by legal rights called
as Breeders rights.
Offsprings of the Breeder seed which can be clearly traced to Breeder seed are called
Foundation Seeds. They are further breaded to give rise to certified seeds. The
production of the foundation seeds must be acceptable to a certification agency.
The progeny of the foundation seed approved and certified by a certifying agency and is
suitable to produce certified seeds is called Registered seed.
This is the last stage, which actually reaches to a farmer. Certified seed is the progeny of
foundation seed and must meet the standards of seed certification prescribed in the
Indian Minimum Seeds Certification Standards, 1988.
Q.77) Consider the following
1. Krishna Circuit
2. Buddhist circuit
3. Mahabharat Circuit
Which of the above circuits are included in the Swadesh Darshan scheme?
a) All
b) 1 and 2 only
c) 2 and 3 only
d) 1 and 3 only

www.iasbaba.com

44

Geography- Subject wise compilation of 60 Day Plan- 2016

Q.77) Solution (b)


Under the Swadesh Darshan scheme, thirteen thematic circuits have been identified, for
development namely: North-East India Circuit, Buddhist Circuit, Himalayan Circuit, Coastal
Circuit, Krishna Circuit, Desert Circuit, Tribal Circuit, Eco Circuit, Wildlife Circuit, Rural Circuit,
Spiritual Circuit, Ramayana Circuit and Heritage Circuit.
Q.78) Consider the following statements regarding Keoladeo National Park:
1. It was formerly known as Bharatpur Bird Sanctuary.
2. It is a natural wetland.
3. It receives thousands of migratory species of birds especially during summer season.
4. It is a World Heritage Site.
Which of the above statements are correct?
a) 1,2 and 3
b) 2 and 3
c) 1 and 4
d) 1,2 and 4
Q.78) Solution (c)
Keoladeo National Park or Keoladeo Ghana National Park formerly known as the Bharatpur
Bird Sanctuary in Bharatpur, Rajasthan, India is a famous avifauna sanctuary that hosts
thousands of birds, especially during the winter season.
It was declared a protected sanctuary in 1971. It is also a World Heritage Site.
Keoladeo Ghana National Park is a man-made and man-managed wetland and one of the
national parks of India. The reserve protects Bharatpur from frequent floods, provides
grazing grounds for village cattle, and earlier was primarily used as a waterfowl hunting
ground.
The sanctuary is one of the richest bird areas in the world and is known for nesting of
resident birds and visiting migratory birds including water birds. The rare Siberian cranes
used to winter in this park but this central population is now extinct. According to founder
of the World Wildlife Fund Peter Scott, Keoladeo National Park is one of the worlds best
bird areas.
Q.79) Despite the efforts of the Forest Department and the Customs Department, the
smuggling of Red Sanders is going unabated. Which of the following statements are
correct about Red Sanders?
1. It is endemic to Southern Eastern Ghats of South India.
2. The tree has characteristic rich red colour wood.

www.iasbaba.com

45

Geography- Subject wise compilation of 60 Day Plan- 2016


3. The wood of the tree is aromatic.
4. It is listed as an endangered species by IUCN.
Select the correct code from the following:
a)
b)
c)
d)

1,3 and 4
1,2 and 3
1,2 and 4
All of the above

Q.79) Solution (c)


Pterocarpus santalinus, with the common names red sanders, is a species of Pterocarpus
endemic to the southern Eastern Ghats mountain range of South India. This tree is valued
for the rich red color of its wood. The wood is not aromatic. The tree is not to be confused
with the aromatic Santalum sandalwood trees that grow natively in South India.
Pterocarpus santalinus is listed as an Endangered species by the IUCN, because of
overexploitation for its timber in South India.
Q.80) Desert vegetation, also called Xerophytes, have adjusted or adapted to its harsh
environment. Which of the following statements are correct adaptations of Xerophytes?
1. Some plants called Succulents, store water in their stems and leaves.
2. Leaves have turned into thorns to reduce water loss and also act as a deterrent to
herbivores.
3. Flowers open at night to lure pollinators.
4. Growth of desert plants is very slow, so that they require less energy.
Select the code from the following:
a) 1,2 and 3
b) 2,3 and 4
c) 1,3 and 4
d) All of the above
Q.80) Solution (d)
Desert Plant Adaptations
Some plants, called succulents, store water in their stems or leaves;
Some plants have no leaves or small seasonal leaves that only grow after it rains. The
lack of leaves helps reduce water loss during photosynthesis. Leafless plants conduct
photosynthesis in their green stems.
Long root systems spread out wide or go deep into the ground to absorb water;
Some plants have a short life cycle, germinating in response to rain, growing, flowering,
and dying within one year. These plants can evade drought.

www.iasbaba.com

46

Geography- Subject wise compilation of 60 Day Plan- 2016

Leaves with hair help shade the plant, reducing water loss. Other plants have leaves that
turn throughout the day to expose a minimum surface area to the heat.
Spines to discourage animals from eating plants for water;
Waxy coating on stems and leaves help reduce water loss.
Flowers that open at night lure pollinators who are more likely to be active during the
cooler night.
Slower growing requires less energy. The plants don't have to make as much food and
therefore do not lose as much water.

Q.81) Consider the following statements regarding epiphytes:


1. These plants need help of a host plant to grow.
2. These plants are parasites and obtain their nutrients from the host plants.
Which of the above statements are incorrect?
a) 1 only
b) 2 only
c) Both 1 and 2
d) Neither 1 nor 2
Q.81) Solution (b)
Epiphytes are the plants growing on host plants but not nourished by the host plants. They
do not draw food from the host plant they only take support in getting access to light. Eg.
Orchids
Q.82) Consider the following statements regarding deciduous trees:
1. They lose all their leaves for a part of the year.
2. In cold climates, trees shed their leaves during summers, to reduce loss of water through
transpiration.
Which of the above statements are correct?
a) 1 only
b) 2 only
c) Both 1 and 2
d) Neither 1 nor 2
Q.82) Solution (a)
In cold climates trees lose their leaves during autumn so that they are bare throughout the
winter.
In hot and dry climates deciduous trees lose their leaves during summer.

www.iasbaba.com

47

Geography- Subject wise compilation of 60 Day Plan- 2016


Q.83) Sea grasses are specialized angiosperms that resemble grass in appearance. They
are the only group of higher plants adapted to life in the salt water. Consider the following
regarding sea grasses.
1. They are found on deep sea floors.
2. There is rich growth of sea grass along Tamilnadu coast and Lakshadweep islands.
3. They help to stabilize bottom sediments to control erosion.
4. Dugong is dependent on sea grass for food.
Which of the above statements are correct?
a) 1,2 and 3
b) 2,3 and 4
c) 1,3 and 4
d) All of the above
Q.83) Solution (b)
They grow in shallow coastal water with sandy or muddy bottoms and require
comparatively calm seas.
IUCN has accorded high priority for conservation of sea grass.
Q.84) Consider the following statements:
1. National parks enjoy a greater degree of protection than wildlife sanctuary.
2. Activities like grazing of livestock, collection of firewood are regulated in a wild life
sanctuary while it is prohibited in a national park.
3. National parks can be created for specific species while wildlife sanctuary is not
particularly focused on any specie.
Which of the above statements are correct?
a) 1 and 2
b) 2 and 3
c) 1 and 3
d) All of the above
Q.84) Solution (a)
Wildlife sanctuaries can be created for specific species while National Park is not focused on
any particular specie.
Doubt: how is the last statement false? I thought that Gir National park and jim corbett
national park were formed for lion and tiger conservation.
please explain

www.iasbaba.com

48

Geography- Subject wise compilation of 60 Day Plan- 2016


A National Park is a broader term. If a particular specie has to be protected in a National
Park, a wild life sanctuary is made inside the national Park itself. Eg. Jim Corbett National
Park has several wildlife sanctuaries to protect tiger.
Q.85) Consider the following statements regarding Topical Evergreen forest in India:
1. These forests grow in areas where rainfall is more than 200cm.
2. The soil of these forests is highly fertile with large humus content.
3. Teak and Sal are important tree species.
4. There is low undergrowth as plant species struggle for sunlight.
Which of the above statements are incorrect?
a) 2,3 and 4
b) 1 only
c) 2 and 3
d) 1 and 4
Q.85) Solution (c)
The soil of tropical rain forest is not fertile as most of the nutrients are leached by heavy
rain.
Teak and Sal are important species of deciduous forest.
The soil of tropical rain forest is not fertile as most of the nutrients are leached by heavy
rain.
Teak and Sal are important species of deciduous forest.
Here in ncert, it says in evergreen forest have luxuriant growth of all the layers. Then , in
question number 8 , the answer shud have been A. Somebody correct me if I'm wrong.
Would be of grt help. :-)
The 4th statement is correct. There is a layered structure in the forest because species try to
fight for life. However, on the floor, very less sunlight reaches so there is very scarce growth
on the floor. a very good source to know about geography and phenomenon, is a BBC series
called 'Planet Earth'. Watch the episode 'the jungles'. There are a total of 11 episodes in the
series.
Q.86) Consider the following statements regarding Onge tribe of India:
1. It is an indigenous tribe of Andaman Islands.
2. They belong to Negrito race.
3. Only less than 100 tribesmen are left.
Which of the above statements are correct?
a) 1 and 2

www.iasbaba.com

49

Geography- Subject wise compilation of 60 Day Plan- 2016


b) 2 only
c) 2 and 3
d) All of the above
Q.86) Solution (d)
The Onge are one of the Andamanese indigenous peoples of the Andaman Islands.
Traditionally hunter-gatherers, they are a designated Scheduled Tribe of India.
Jarawa and Onge are endangered tribes of Andaman.
This question again initiated a lot of discussion about the population of the Onge Tribe. Well
the purpose of this question was not to check whether you know about the exact number of
the tribe or not. It was only to tell that Onge Tribe is important with examination point of
view.
However, the data which we saw while making question was old. And many references
given by you are latest which tells that Onge tribe is has got more than 100 members now.
Well, good for them and good for us that our efforts to protect them are showing good
results.
Lets hope UPSC wont get into the population of any tribe.
Now with the kind of discussion that we had on Discuss, we do pray that we get a question
on Onge Tribe. The BABA people will get one Bonus question.
Q.87) Consider the following statements:
1. On the basis of endangerment, UNESCO has categorized the languages into two
categories.
2. According to UNESCO, a language becomes extinct when nobody speaks or remembers
the language.
3. Protection and Preservation of Endangered Languages of India is a central government
scheme to save the languages spoken by less than 10,000 people in India.
Which of the above statements are correct?
a) 1 and 2
b) 2 and 3
c) 1 and 3
d) All of the above
Q.87) Solution (b)
UNESCO has categorised languages on basis of endangerment as: vulnerable, definitely
endangered, severely endangered and critically endangered.
According to the criteria adopted by UNESCO, a language becomes extinct when nobody
speaks or remembers the language.

www.iasbaba.com

50

Geography- Subject wise compilation of 60 Day Plan- 2016


The government has initiated a scheme to protect, preserve and document endangered
Indian languages - spoken by less than 10,000 people.
Under the scheme "Protection and Preservation of Endangered Languages of India",
Mysore-based Central Institute of Indian Languages (CIIL) is working on the protection,
preservation and documentation of endangered languages.
Q.88) Consider the statements regarding a particular valley which was in the news
recently.
1. It is a part of Gangotri National Park
2. It is a cold desert and looks like a replica of Tibetan plateau
3. It is home to Snow Leopard
Select the correct code
a) Nellong valley
b) Kashmir valley
c) Alaknanda valley
d) Bhagirathi valley
Q.88) Solution (a)
http://www.hindustantimes.com/travel/nelong-valley-another-ladakh-tucked-in-theuttarakhand-himalayas/story-42e5ckQmuHERYvTJ6SVtmL.html
Q.89) Central Zoo Authority, a body set up in 1992 for the oversight of zoos.
Consider the following statements regarding CZA.
1. It is set up under the Wild Life Protection Act, 1972.
2. It is an affiliated member of World Association of Zoos and Aquarium.
3. It doesnt have the power to regulate the trade of endangered species.
Choose the correct statements from the following code.
a) 1 and 2 only
b) 2 and 3 only
c) 1 and 3 only
d) All the above
Q.89) Solution (a)
It does have the power to regulate the trade of endangered species. Apart from that it also
has the power to regulate and approve the exchange of animals between Indian and foreign
zoos.

www.iasbaba.com

51

Geography- Subject wise compilation of 60 Day Plan- 2016


Q.90) Himalayan yew is a plant found in Himachal Pradesh and Arunachal Pradesh, was in
news recently.
Consider the following statements regarding the plant.
1. It is a medicinal plant.
2. A chemical compound called taxol obtained from the plant is used as an effective anticancer drug.
Choose the incorrect statements from code below.
a) 1 only
b) 2 only
c) Both
d) None of the above
Q.90) Solution (d)
A chemical compound called taxol obtained from the plant is used as an effective anticancer drug.
http://www.downtoearth.org.in/news/himalayan-yew-to-fight-cancer-29559
Q.91) Consider the following
1. Trees shed their leaves in the dry season to conserve water
2. These are economically most important forests of India
3. Sal, Teak and Neem are some of the important trees of this forest
Identify the forest?
a) Deciduous Forests
b) Evergreen Forests
c) Tropical Deciduous Forests
d) Temperate Deciduous Forests
Q.91) Solution (c)
Tropical deciduous are the monsoon forests found in the large part of India, northern
Australia and in Central America. These regions experience seasonal changes. Trees shed
their leaves in the dry season to conserve water.
In a large part of our country we have this type of forest. These forests are also called
Monsoon Forests. They are less dense. They shed their leaves at a particular time of the
year in spring and at onset of summer. Important trees of these forests are sal, teak,
peepal, neem and shisham. (The names that we hear often) They are found in Madhya
Pradesh, Uttar Pradesh, Bihar, Jharkhand, Chhattisgarh, Orissa, and in parts of Maharashtra.
Tropical Deciduous Forests are economically most important forests of India.

www.iasbaba.com

52

Geography- Subject wise compilation of 60 Day Plan- 2016


Q.92) If you are travelling through Eastern Himalayas, which of the following you will
encounter?
1. Golden Langur
2. Takin
3. Black Bear
Select the correct code
a) 1 and 2
b) 2 and 3
c) Only 2
d) 1, 2 and 3
Q.92) Solution (d)
Q.93) Consider the following statements
1. The leaves of these forest falls in autumn season
2. They are found in Temperate Zone
3. These forests in China and Europe are basically man made
What is the name of this forest?
a) Coniferous Forest
b) Deciduous Forest
c) Tropical Rain Forest
d) None
Q.93) Solution (b)
A deciduous forest is one where the leaves fall from the trees in autumn. Deciduous means
"falling out or off at a certain season". he average temperature is 50 degrees and the
average rainfall is 30 to 60 inches per year. These forests are located in the temperate zone
above the tropical forests and below the coniferous forests. These forests have their own
specific plants, trees, animals, and shrubs. Deciduous forests are found in the eastern part
of North America and the middle of Europe. Asia, southwest Russia, Japan, and eastern
China also have these forests. All of the deciduous forests in America are second growth.
This means all the original trees have been cut and the forests have grown from seeds and
seedlings. Both China and Europe have cleared all of their first growth deciduous forests for
agriculture. All of the deciduous forests in Europe and China are man-made.
Q.94) Consider the following statements based on UN report on E-waste
1. The bulk of global e-waste constitutes from mobile phones, calculators, personal
computers, printers, and small information technology equipment.
2. The lowest amount of e-waste per inhabitant was generated in South Asian countries

www.iasbaba.com

53

Geography- Subject wise compilation of 60 Day Plan- 2016


3. China is the largest producer of e-waste in the world
Select the correct code/s
a) 1, 2 and 3
b) 1 and 2
c) 2 and 3
d) None
Q.94) Solution (d)
While only 7 per cent of e-waste last year was made up of mobile phones, calculators,
personal computers, printers, and small information technology equipment, almost 60 per
cent was a mix of large and small equipment used in homes and businesses, such as vacuum
cleaners, toasters, electric shavers, video cameras, washing machines, electric stoves,
mobile phones, calculators, personal computers, and lamps.
The lowest amount of e-waste per inhabitant was generated in Africa (1.7 kg/inhabitant).
The continent generated 1.9 Mt of e-waste in total.
India is the fifth biggest producer of e-waste in the world, discarding 1.7 million tonnes (Mt)
of electronic and electrical equipment in 2014, a UN report has warned that the volume of
global e-waste is likely to rise by 21 per cent in next three years.
The Global E-Waste Monitor 2014, compiled by U.N.s think tank United Nations University
(UNU), said at 32 per cent, the U.S. and China produced the most e-waste overall in 2014.
India is behind the U.S., China, Japan and Germany.
Q.95) Consider the following with respect to Western Ghat and rainfall pattern along it
1. Karnataka receives less rainfall than Kerala and Maharashtra because of steep slopes
and continuous Mountains
2. The mountain topography in Karnataka is broader than the narrow topography of the
Ghats in Maharashtra.
Select the incorrect statements
a) Only 1
b) Only 2
c) Both
d) None
Q.95) Solution (a)
Kindly understand that mountains of Karnataka are not steep sloping but gentle sloping.
So, 1st statement is incorrect.

www.iasbaba.com

54

Geography- Subject wise compilation of 60 Day Plan- 2016


The mountain topography in Karnataka is broader than the narrow topography of the
Ghats in Maharashtra. Due to the greater width of the mountains, the rain bearing winds
have to necessarily travel a longer distance and have more time for the drops to coalesce
and precipitate as rainfall, resulting in higher rainfall. In contrast, the narrow width of the
Ghats in Maharashtra allows the rain-bearing wind to cross over to the leeward side rapidly
before precipitation can occur. As for Kerala, the Ghats there are in the form of isolated
mountains, where the rain-bearing winds can easily cross over to the leeward side through
the gaps in between without precipitation occurring.
Second, the slope of the mountain has a direct bearing on the possibility of precipitation.
This is borne out by the Ghats of Karnataka where the mountains are gently sloping,
compared to the steep slopes of the Ghats in Maharashtra and Kerala.
The air parcel will retain its energy and speed for a longer time when the slope is gradual.
This will provide sufficient vertical motion to cloud droplets to grow by collision
coalescence process and hence form precipitation.
Third, the gentle slope provides a greater area for sunlight absorption and heating leading
to greater convection when compared with an abrupt slope i.e. less Ghat area such as that
of the Maharashtra and Kerala Ghats.
Fourth, the continuous mountain range presents a greater barrier to rain-bearing winds
than a range comprising isolated mountains with gaps in between where the winds can
easily pass to the leeward side. Unlike in the case of Kerala, the Ghats in Maharashtra and
Karnataka are continuous.
Q.96) Indian Green Building Council (IGBC) New Green Buildings rating system addresses
green features under certain categories. What are they?
1. Site selection and planning
2. Building Materials and Resources
3. Innovation and Development
4. Water Conservation
5. Noise Conservation
6. Energy Efficiency
Select the correct order
a) 1, 3, 4 and 5
b) 1, 2, 3, 4, 5 and 6
c) 1, 2, 3 and 4
d) 1, 2, 3, 4 and 6
Q.96) Solution (d)

www.iasbaba.com

55

Geography- Subject wise compilation of 60 Day Plan- 2016

IGBC Green New Buildings rating system addresses green features under the following
categories:
Sustainable Architecture and Design
Site Selection and Planning
Water Conservation
Energy Efficiency
Building Materials and Resources
Indoor Environmental Quality
Innovation and Development

Q.97) What are the factors for extinction of wildlife throughout the world?
1. Alteration of habitat
2. Introduction of foreign species
3. Control of pests and predators
4. Captive breeding
Select the correct code:
a) 1 and 2 Only
b) 1, 2 and 3 Only
c) 1, 3 and 4 Only
d) All of the above
Q.97) Solution (b)
Predators or pests are important biotic components of the ecosystem. Natural predators
make the prey population stronger by removing weak members of the prey population but
human beings generally remove the strongest specimens. Captive breeding is for
conservation of troubled species (a measure to safeguard wildlife)
Babaji- introduction of foreign species should not necessarily be a factor for extinction.
Don't you think it should be only an invasive species which threatens the existence of the
prevailing animals?
a series of argument followed after this comment, About how a foreign specie can be the
cause of extinction.
As a matter of fact, introduction of foreign species are one of the major cause of extinction.
They become hazardous to indigenous species. extinction of local species can be caused by If foreign specie is a predator and indigenous specie is not evolved enough to have a
defence mechanism against this new enemy.
Foreign species can be invasive. Especially many plant species can replace the
indigenous vegetation causing extinction.
Foreign species may introduce foreign pathogens. Indigenous species may not have an
immunity against them and they can cause mass destruction.

www.iasbaba.com

56

Geography- Subject wise compilation of 60 Day Plan- 2016

I hope things are clear now.


Another One regarding Pest and Predators- the statement talks about control of Pest and
Predators by the humans.
Usually the predator population controls the Prey population and vice versa. Taking one
away from the equation will hamper the existence of the other.
Q.98) Consider the following statements regarding Halleys Comet:
1. It is visible on earth after every 76 years.
2. It revolves around the sun in elliptical orbit.
3. As it gets closer to the sun, it starts glowing due to burning of its particles.
Which of the above statements are correct?
a) 1 and 2
b) 2 and 3
c) 1 and 3
d) All of the above
Q.98) Solution (d)
Halley's Comet takes 75 to 76 years to complete an orbit around the Sun. Comets travel in
elliptical orbits, usually going from a great distance away from the Sun (hundreds of millions
of miles) to relatively close (just a few million miles) to it. As the get closer Sun, they begin
glowing is the Sun burns off its particles. This makes them visible from Earth, and every 75
to 76 years, as Halley's Comet approaches the Sun, it becomes visible in the night sky,
looking a bit like a smudged star.
Q.99) The Gravitational force between the two bodies is :
a) Directly proportional to their masses and inversely proportional to the distance between
them
b) Directly proportional to the distance between them and inversely proportional to their
masses
c) Directly proportional to the masses and distance between them
d) Inversely proportional to the masses and distance between them
Q.99) Solution (a)
The Gravitational force between the two bodies is Directly proportional to the product of
Masses of the two bodies. And inversely proportional to the square of distance between
them.
The gravitational force increases with increase in the mass and decrease with the increase in
distance between the two bodies.
Q.100) Communication satellites are usually launched in:

www.iasbaba.com

57

Geography- Subject wise compilation of 60 Day Plan- 2016


a)
b)
c)
d)

Polar orbit
Lunar orbit
Geostationary Orbit
No fixed orbit. They are moved as desired.

Q.100) Solution (c)


Geostationary satellites have a geostationary orbit (GEO), which is 35,786 kilometres
(22,236 mi) from Earths surface. This orbit has the special characteristic that the apparent
position of the satellite in the sky when viewed by a ground observer does not change, the
satellite appears to "stand still" in the sky. This is because the satellite's orbital period is the
same as the rotation rate of the Earth. The advantage of this orbit is that ground antennas
do not have to track the satellite across the sky, they can be fixed to point at the location in
the sky the satellite appears.
Q.101) As seen from the Earth, a solar eclipse is a type of eclipse that occurs when the
Moon passes between the Sun and Earth, and the Moon fully or partially blocks
("occults") the Sun. which of the following statements are incorrect about Solar Eclipse?
1. It can occur only at full moon when sun and moon are at conjunction.
2. The alignment of sun, moon and earth respectively in a straight line is called syzygy.
3. During total solar eclipse, the complete moons shadow is called Penumbra.
Select the code from the following:
a) 1 only
b) 2 and 3
c) 1 and 3
d) All of the above
Q.101) Solution (b)
As seen from the Earth, a solar eclipse is a type of eclipse that occurs when the Moon
passes between the Sun and Earth, and the Moon fully or partially blocks ("occults") the
Sun. This can happen only at new moon, when the Sun and the Moon are in conjunction as
seen from Earth in an alignment referred to as syzygy. In a total eclipse, the disk of the Sun
is fully obscured by the Moon. In partial and annular eclipses, only part of the Sun is
obscured.
During Total Solar eclipse, the complete moons shadow is called Umbra.
*Note: Question corrected as it was wrongly given as select the correct, rather it should be
select the incorrect.

Q.102) The Sun is still extremely hot, whereas the planets are much cooler. Which of the
answers gives the correct explanation?

www.iasbaba.com

58

Geography- Subject wise compilation of 60 Day Plan- 2016


a) The Sun burns hydrogen in its interior, whereas the planets supply of hydrogen has run
out.
b) The Sun fuses hydrogen to helium, releasing energy, whereas the planets supply of
hydrogen is less.
c) The Sun produces energy by fission reactions, whereas the planets were never hot
enough to produce energy this way.
d) It is too cold deep inside the planets to have fusion reactions necessary to produce
energy from hydrogen.
Q.102) Solution (b)
Most planets contain very little hydrogen, but where there is large amounts present; such as
in Jupiter, the temperature is not high enough to initiate and sustain nuclear reactions.
Q.103) Two moons of the same mass orbit a planet. The orbit of the Earth's Moon is
almost circular, but the orbits of the moons are much more complicated. What is the most
likely reason for this?
a) The gravitational pull of the planet varies, changing their otherwise normal orbits.
b) The moons attract each other when they are close together, changing their normal
orbits.
c) The inner moon has a circular orbit, affecting the other moon's orbit.
d) The outer moon has a circular orbit, affecting the other moon's orbit.
Q.103) Solution (b)
The gravitational force changes as two bodies get closer to each other. As the two moons
comes closer to each other, the net force changes making the path of the moon complex.
Q.104) Consider the following statements regarding satellite that has been placed in a low
polar orbit:
1. The satellite is ideal to take weather and spying photographs.
2. The satellite takes 24 hours to complete one orbit, which means that it stays above the
same point over the Earth.
3. The satellite orbit's over both poles, whilst the Earth rotates at 90 to this direction
beneath the satellite.
4. Each orbit only takes a few hours, which allows the whole surface of the Earth to be
monitored in one day.
Which of the above statements are NOT correct?
a) 2 only
b) 1,3 and 4
c) 1 and 2
d) 2,3 and 4

www.iasbaba.com

59

Geography- Subject wise compilation of 60 Day Plan- 2016

Q.104) Solution (a)


Geostationary satellite takes 24 hours to complete one orbit and stays above the same point
over earth.
Q.105) Which of the following phases of the Moon occur during a spring tide?
a) Crescent Moon and Gibbous Moon.
b) New Moon and Full Moon.
c) Half Moon.
d) New Moon, Half Moon and Full Moon.
Q.105) Solution (b)
During a spring tide, the Sun, Earth and Moon are all lined up. This happens during New
moon (when moon is placed between sun and earth) and the full moon (when earth is
placed between sun and the moon).
Q.106) Consider the following:
1. The inner core of the earth is liquid while the outer core is solid.
2. The inner core mainly consists of an alloy of Iron, Aluminum and Manganese.
3. The inner and outer core is separated by Lehman Discontinuity.
Which of the above statements are correct?
a) 1 and 3
b) 1 and 2
c) 3 only
d) All of the above
Q.106) Solution (c)
The Earth's inner core is the Earth's innermost part and according to seismological studies, it
has been believed to be primarily a solid ball with a radius of about 1220 kilometers, or 768
miles (about 70% of the Moon's radius). It is composed of an ironnickel alloy and some
light elements. The temperature at the inner core boundary is approximately 5700 K.
The outer core was determined to be liquid from observations showing that compressional
waves pass through it, but elastic shear waves do not or do so only very weakly.
Q.107) The Earth's crust is an extremely thin layer of rock that makes up the outermost
solid shell of our planet. It amounts to less than half of 1 percent of the planet's total mass
but plays a vital role in most of Earth's natural cycles. Which of the statements are correct
about Earths crust?
1. Continental crust is thicker than the oceanic crust.

www.iasbaba.com

60

Geography- Subject wise compilation of 60 Day Plan- 2016


2. Continental Crust, made up of granitic rock, is denser than the oceanic crust which is
made up of Basaltic rock.
3. During Subduction, the continental crust, being denser, plunges under the oceanic crust.
Select the code from below:
a) 1 only
b) 1 and 2
c) 2 and 3
d) All of the above
Q.107) Solution (a)
Continental crust is much thicker than Oceanic crust. Continental crust is around 40 km thick
on average and can reach up to 70-80 km thickness under high mountain chains. Oceanic
crust is around 8-10km thick.
Oceanic crust made of basalt is denser than continental crust. It plunges under continental
crust during continental-oceanic convergence. This process is called subduction.
Q.108) Consider the following statement:
1. Endogenetic forces are responsible for the formation of secondary landforms.
2. The energy emanating from within the earth is the main force behind endogenic
geomorphic processes.
3. The energy in the interior of earth is due to radioactive decay and primordial heat from
the origin of the earth.
Which of the above statements are correct?
a) 1 and 2
b) 2 and 3
c) 1 and 3
d) All of the above
Q.108) Solution (d)
There are three types of landforms- Primary, secondary and tertiary.
Primary landforms are- continents and oceans
Secondary- folded mountains, volcanoes, mid-oceanic ridges etc.
Tertiary- v-shaped valleys, meanders, inselbergs, mushroom rocks etc.
Primary and secondary landforms are formed by endogenetic forces.
Statement 2 and 3 are self explanatory.
Q.109) Consider the following statements:

www.iasbaba.com

61

Geography- Subject wise compilation of 60 Day Plan- 2016


1. Orogeny is a mountain building process whereas epeirogeny is continental building
process.
2. Endogenetic forces act in horizontal direction in orogeny while they act in vertical
direction in epeirogeny.
Which of the above statements are incorrect?
a) 1 only
b) 2 only
c) Both 1 and 2
d) Neither 1 nor 2
Q.109) Solution (d)
Both the statements are correct.
Orogeny and epeirogeny are both endogenetic forces. Epeirogeny acts in vertical direction
pushing the whole continental landmass up or down. It is an extremely slow process.
Orogenetic force acts in horizontal direction, pushing the lithosphere plates. When these
plates interact with each other, landforms like folds and faults are formed.
Q.110) Which statement best describes motion on a normal fault?
a) The hanging wall block moved downward relative to the footwall block.
b) The hanging wall block moved upward relative to the footwall block.
c) The hanging wall block moved horizontally relative to the footwall block.
d) Faults are simple breaks in the crust along which no movement has occurred.
Q.110) Solution (a)

This is the diagram of a normal fault. Due to tangential stress, the fault is created with
hanging wall block sliding downwards forming a scarp on footwall.
Q.111) Which of the following structures are the example of Aeolian landforms?
1. Mushroom rocks
2. Barchans
3. Cascades
4. Gorge
5. Gully

www.iasbaba.com

62

Geography- Subject wise compilation of 60 Day Plan- 2016


Select the code from below:
a) 1 and 2
b) 3,4 and 5
c) 1,2 and 5
d) 2,3 and 4
Q.111) Solution (a)
Aeolian land forms are found in regions of the Earth where erosion and deposition by wind
are the dominant geomorphic forces shaping the face of the earth. They are found usually in
dry places.
Mushroom rocks are found in rocky deserts where winds keep cutting the base of a rock and
the shape of the rock looks like a mushroom.
Barchans are large sand dunes found in Sandy deserts like Sahara, Dasht e Kavir etc.
Rest are the example of fluvial landforms. i.e. landforms created by running water.
Q.112) Consider the following statements:
1. Intrusive igneous rocks have larger crystal grains than extrusive igneous rocks.
2. Igneous rocks are always crystalline.
3. Igneous rocks are main source of fossils.
Which of the above statements are incorrect?
a) 1 only
b) 2 and 3
c) All of the above
d) None of the above
Q.112) Solution (b)
Crystal formation depends on the rate of cooling of the rock. Since intrusive rock cools
slowly, their crystals are well developed and have larger grains. Extrusive rocks are suddenly
cooled hence they have smaller crystal grains.
Igneous rocks can be both crystalline and amorphous.
Fossils are not found in igneous rocks, hot lava burns all the organic content that comes in
its way. Fossils are most commonly found in sedimentary rocks. If these sedimentary rocks
are metamorphosised then sometimes they are found in metamorphic rocks too.
Q.113) Which among the following is the first ever inter-State river linking project since
Indias independence?
a) Chambal-Yamuna project
b) Mahanadi-Godavari project
c) Par-Tapi- Narmada project

www.iasbaba.com

63

Geography- Subject wise compilation of 60 Day Plan- 2016


d) Ken-Betwa project
Q.113) Solution (d)
Explanation:
Ken-Betwa project is the first-ever inter-State river linking project since Indias
independence
The main feature of the project is a 230-km long canal connecting the Ken and Betwa
rivers which will irrigate 3.5 lakh hectares of drought-prone Bundelkhand
Q.114) Which among the following is/are true about Djibouti?
1) Djibouti islands is located between Gulf of Suez and Red Sea
2) A military base in Djibouti along with major port development projects in Myanmar and
Sri Lanka are defining the contours of Chinas Maritime Silk Road
3) The Horn of Africa nation is strategically located on the junction of the Indian Ocean and
the Red Sea
Choose the appropriate option:
a) 1 and 2
b) 2 and 3
c) 1 and 3
d) 1, 2 and 3
Q.114) Solution (b)
Explanation:
Djibouti islands is located between Gulf of Aden and Red Sea (not Gulf of Suez)

Q.115) Arrange the following states according to first arrival of Southwest Monsoon
rainfall to last to receive:
1) Kerala
2) Meghalaya
3) Andhra Pradesh
4) Bihar

www.iasbaba.com

64

Geography- Subject wise compilation of 60 Day Plan- 2016

Choose the appropriate code from below:


a) 2-4-3-1
b) 1-3-2-4
c) 2-3-4-1
d) 1-2-3-4
Q.115) Solution (d)

Q. 116) Consider the following National Parks and their respective locations
National Park
Location
1. Balphakram
Meghalaya
2. Bison
Mizoram
3. Gorumara
West Bengal

www.iasbaba.com

65

Geography- Subject wise compilation of 60 Day Plan- 2016


4.
5.

Mollem
Nagarhole

Andaman and Nicobar


Karnataka

Select the incorrectly matched


a) 1 and 4
b) 2 only
c) 2 and 4
d) 1 and 2
Q. 116) Solution (c)
National Park
1. Balphakram
2. Bison
3. Gorumara
4. Mollem
5. Nagarhole
6. MG Marine NP
7. Murlen

Location
Meghalaya
Tripura
West Bengal
Goa
Karnataka
A&N islands
Mizoram

Q. 117) Arrange the following states in decreasing order in context of their boundaries
shared by other states/union territories.
1- Maharashtra, 2-Madhya Pradesh,3-Assam, 4-Uttar Pradesh,5-Punjab
a)
b)
c)
d)

41325
43512
43125
34125

Q. 117) Solution (c)


1- Maharashtra
2- Madhya Pradesh
3- Assam
4- Uttar Pradesh
Rajasthan
5- Punjab

Gujarat, MP, CHattisgarh, Telangana, Karnataka, Goa, DN&H


Maharashtra, Gujarat, Rajasthan, UP, Chattisgarh
Ar. Pradesh, WB, Megahalaya, Tripura, Mizoram, Manipur, Nagaland
NCR, Haryana, Uttranchal, Bihar, Jharkhand, MP, Chattisgarh,
Himachal Pradesh, Haryana, Rajasthan

Q. 118) Arrange the following passes in north to south order


1- Burzil, 2-Rohtang, 3-Mohan, 4-Pensi La, 5-Zoji La, 6-Shipki La
a) 1-4-5-2-6-3
b) 1-3-4-5-2-6

www.iasbaba.com

66

Geography- Subject wise compilation of 60 Day Plan- 2016


c) 1-5-4-2-6-3
d) 1-4-3-2-5-6
Q. 118) Solution (c)
Brurzil- Srinagar and Gilgit, J&K
Rohtang- Pir Panjal range, Himachal Pradesh
Mohan- Shiwalik hills, Sikkim
Zojila- ladakh, J&K
Shipki la- India-China border post, Himachal Pradesh

www.iasbaba.com

67

Вам также может понравиться